Sunteți pe pagina 1din 76

GREAT IS 2019. WE PREVAIL.

MDSC 1001 Environment and Health


.
Problem 1
Cellular Worlds

In viewing a specimen of animal cells through a light microscope, two first-year


veterinary medicine students, Shrinath and Peters, were intrigued by the various
compartments within the cells. Shrinath was interested in what could have been seen with
a more powerful microscope and also wondered why biochemists needed to mash up
cells in order to study their behaviour. Doesnt that kill themwouldnt they learn more
if the studied the whole cell while it was alive? he said.

OBJECTIVES
1. Describe the ultrastructure of an animal cell.
2. Describe the structure of the cell membrane.
3. Describe the basic structure and properties of lipids.
4. Relate the morphology and relative abundance of various cellular organelles
to the functions of the cell.
5. Discuss the need for intra-cellular compartmentalisation and the basic
principles of intracellular metabolism.
6. Discuss the different methods used in the study of cells and the preparation of
tissues.

January 7, 2008
GREAT IS 2019. WE PREVAIL.

Problem 2
Is Street Food Safe?

An article in the local newspaper recently reported unusually high numbers of cases of
diarrhoea over the last three months. An expert medical source in the newspaper report
suggested that it could be an outbreak of cholera, an intestinal infection caused by
bacteria and spread through the consumption of contaminated water and food. The article
indicated that about 10-20% of those infected would develop severe watery diarrhoea
with vomiting. This secretory diarrhoea is a result of cholera toxin binding to the
epithelial lining of the small intestine and triggering a series of events culminating in the
massive efflux of electrolytes and water into the intestinal cavity that can lead to severe
dehydration and death. Citizens were being advised to seek medical treatment promptly,
boil their drinking water and be mindful when buying food from street vendors. Public
Health Authorities were undertaking investigations and moves were being made to clamp
down on vendors who did not display valid food badges.

OBJECTIVES
1. Review the fluid mosaic model of the plasma membrane of eukaryotic cells.
2. Describe how water is distributed in the various body compartments.
3. List the different types of diarrhoea and their physiological causes.
4. Explain the mechanism of action of cholera toxin.
5. Discuss the different types of dehydration and indicate which type cholera
causes.
6. Describe the causes and consequences of dehydration of the fluid
compartments.
7. Discuss the use of Darrow-Yannet diagrams.
8. Discuss the use of epidemiology to investigate public health problems.

January 7, 2008
GREAT IS 2019. WE PREVAIL.

Problem 3
Calming the Nerve

A novel compound was being tested for its effect on a nerve preparation. Intracellular
recordings showed expected changes in membrane potentials following changes in the
ionic composition of the extracellular fluid. After application of the novel compound the
nerve no longer generated an action potential when stimulated.

OBJECTIVES
1. Briefly describe the structure of a typical neuron.
2. List the major anions and cations found in extracellular and intracellular fluid.
3. Explain the processes that maintain the resting membrane potential of cells.
4. Describe how changes in the ionic composition (Na+, K+ & Cl-) of
extracellular fluid affect the polarisation of excitable membranes.
5. Explain how action potentials are generated in a nerve.
6. Discuss compounds that can affect action potential generation.

January 7, 2008
GREAT IS 2019. WE PREVAIL.

Problem 4
A Chain is as Strong as its Weakest Link

Six-year-old Jennifer was taken to the doctor because her parents were worried about the
fact that she always complained of being tired and often had mild fever. Recently her
mother had noticed that her eyes looked slightly yellow. After receiving the results of the
blood test she had ordered, the doctor told her parents that one of the proteins in
Jennifers red blood cells was defective. She explained that this was caused by a gene
mutation that affected only one of the hundreds of amino acid building blocks found in a
particular protein called haemoglobin. Unfortunately, she said, this single substitution
does have a profound effect on the performance of red cells.

OBJECTIVES
1. Discuss the histology of blood cells.
2. Explain what is meant by the terms primary, secondary, tertiary and
quaternary structure of a protein.
3. Explain why the primary structure of a protein is said to determine its three
dimensional structure and its function.
4. Explain why the substitution of a single amino acid alters the shape and
biochemical function of the haemoglobin molecule.
5. Describe the different types of point mutations and their possible effects on
protein structure and function.
6. Discuss the role of epidemiology in identifying this patients diagnosis.
7. Discuss the concepts of health, illness and disease.

January 7, 2008
GREAT IS 2019. WE PREVAIL.

Problem 5
Happy, Grumpy and DROOPY

A 21-year-old salesgirl complained of weakness in the shoulders and drooping eyelids.


These symptoms appeared in the evenings but disappeared after a nights rest. On and off
she also had difficulty swallowing, particularly at dinner time.

Her GP suspected a problem with either her nerves or muscles and referred her to a
neurologist. The latter performed an edrophonium test and EMG (electromyogram) to
confirm the diagnosis.

OBJECTIVES
1. Describe the anatomy of the neuromuscular junction.
2. Explain the process of synaptic transmission at the neuromuscular junction.
3. Identify drugs that can block neuromuscular transmission.
4. Briefly describe the pathophysiology and treatment of myasthenia gravis.
5. Describe how an EMG is performed.
6. Describe three methods that distinguish normal from abnormal tests result in
the diagnosis of this patient.

January 7, 2008
GREAT IS 2019. WE PREVAIL.

Problem 6
Their Legs Went Weak

Mr & Mrs. Singh could hardly digest the news. They had just returned from the doctors
office. He had told them that it was very likely that their son Shiva had an inherited
disorder called Muscular Dystrophy. He spent a lot of time explaining all the tests that
would have to be done to confirm the diagnosis and to determine whether it was
Duchennes Muscular Dystrophy (DMD). All poor Mr and Mrs Singh remembered was
what he had said about Shiva being unlikely to live past the age of 25.

But how did he get it? Mr. Singh asked his wife, I cant remember anyone in our
family having anything like that?

After getting over the initial shock, Mrs. Singh decided to call her niece Nalini who was a
medical student. Nalini explained that DMD was a genetic disorder that only occurred in
boys and that people with DMD did not produce a protein that was necessary for proper
muscle function. She said that Duchennes was only one of several types of muscular
dystrophies and that each type differed in the muscles affected, the age of onset, and its
rate of progress but she was not sure what caused them.

Nalini explained (as simply as she could) the different tests that were likely to be done
like a muscle biopsy, histological tests, immunohistochemistry, EMG and DNA analyses.
She knew that there had been research directed at developing techniques for replacing the
defective gene. She had read about attempts to repair the defective gene in a canine
model of DMD in golden retrievers (GRMD) but wasnt sure whether these gene
therapies were being used on humans yet.

OBJECTIVES
1. Discuss the chromosome theory of inheritance and the genetic basis of
disease.
2. Discuss the inheritance and genetics of DMD.
3. Briefly describe the causes of DMD and other muscular dystrophies.
4. Describe the organization, microscopic anatomy and ultrastructure of skeletal
muscle.
5. Describe the structure of the actin and myosin myofilaments and their
relationship to other proteins important for muscle contraction.
6. Discuss the role of dystrophin.
7. Describe the different types of tests used to diagnose and characterise
muscular dystrophies.
8. Outline the types of DNA analyses that can be used to diagnose and
characterise genetic disorders.
9. Explain what is meant by gene therapy and give a brief account of what
progress has been made towards gene therapy for DMD.
10. Discuss the concept of natural history and determinants of health in relation to
the diagnosis and treatment of this patient.
11. Contrast the epidemiological triangle and web of causation in identifying

January 7, 2008
GREAT IS 2019. WE PREVAIL.

the cause of this disease.

January 7, 2008
GREAT IS 2019. WE PREVAIL.

Problem 7
HIV and Pregnancy

Charmaine, a 23 year old UWI student has just found out that she is pregnant. Only three
weeks before she learned that she was HIV positive and is barely coping. She worries that
others will find out and hasnt even told her family yet. Now she is back in Dr. Williams
office in tears. She wants to keep her baby but is worried about its health.

What is the chance that my baby will be infected? she asks.

Well, there have been many studies, says Dr. Williams. Without treatment,
transmission rates vary widely depending on the country. In the Caribbean it is about
28%. Most of that transmission occurs during labour and via breastfeeding. As long as
you stay relatively healthy, your placenta will help to protect the foetus against infection
during the actual pregnancy. The good news is that the risk at all stages can be
significantly reduced if we use antiviral treatment during pregnancy and delivery and for
the baby after it is born. The recommended drugs are AZT, nevirapine and 3TC - they
interfere with the replication of the viral genome.

Charmaine is so anxious about her babys health she wants to start treatment
immediately, but Dr. Williams tells her she will have to wait until after 14 weeks
gestation.

OBJECTIVES
1. Discuss the main issues that might create problems for a woman diagnosed
with HIV in the Caribbean.
2. Discuss mother-to-child transmission of HIV and prevention strategies in
developed countries versus resource-limited settings.
3. Describe the structure and role of a mature hemochorial placenta.
4. List infective agents that are able to cross the placenta and are teratogenic.
5. Review nucleic acid synthesis and replication.
6. Explain the mechanism of action of antiretroviral drugs (with reference to the
role of reverse transcriptase in the virus genome replication).
7. Explain why drug therapy is usually delayed until after 14 weeks gestation.

January 7, 2008
GREAT IS 2019. WE PREVAIL.

MDSC 1002 Basic Paraclinical Sciences

PROBLEM 1

Tests Performed in Chemical Pathology Laboratory

Mrs. Sandy was offered a position with The University of the West Indies and was
required to submit a Certificate of Medical Fitness. Her general practitioner filled out a
laboratory form requesting a biochemical profile, haematological profile, urine analysis,
HIV screen and a test for the Hepatitis B virus. Mrs. Sandy was puzzled, as she did not
understand the meanings of these medical terms. She, however, went to the laboratory
department of the Eric Williams Medical Sciences Complex and inquired as to the type of
biological specimen required for the tests.

OBJECTIVES:
1. Discuss the important items of information required on a laboratory request form.
2. List the different biochemical investigations done in a Chemical Pathology
Laboratory.
3. Discuss the different types of biological specimens needed for investigations done
in a chemical pathology laboratory.
4. Describe three (3) different factors used for distinguishing normal and
abnormal findings in diagnostic tests.
5. Explain what is meant by screening with regard to HIV and Hepatitis B.

January 7, 2008
GREAT IS 2019. WE PREVAIL.

PROBLEM 2

Blood Components and Normal CBC Values

Kwame, a first-year medical student and a regular blood donor decided to use this
experience to learn something about blood. He asked the technician to obtain a suitable
sample for a CBC. He then went to the blood bank where he saw plasma being separated
from red cells. Later that afternoon, he returned to the hematology lab for his results and
was given a slip with the following values:

Hb 15g/dl; Wbc 8 x 109/l; Platelets 165 x 109/l; MCV 90 fl; Neutrophils 4.5 x 109/l;
Lymphocytes 2.5 x 109/l; Eosinophils 0.3 x 109/l; Basophils 0.6 x 109/l.

OBJECTIVES:
1. Identify normal ranges of hemoglobin by age, sex and physiology state such as
pregnancy.
2. Identify the mean corpuscular volume (MCV) as an important parameter in
indicating possible causes of low hemoglobin.
3. Describe how the cell population would appear under a microscope.
4. Briefly discuss the composition of plasma.
5. List the difference between plasma and serum.
6. Identify the components of whole blood that are available for transfusion.
7. What indications are there for blood transfusion?
8. List and explain normal CBC values.

January 7, 2008
GREAT IS 2019. WE PREVAIL.

PROBLEM 3

Inflammation and Wound Healing

A 12-year old secondary school student lost control of his bike and fell on the road. He
had a lacerated wound on his scalp, superficial and deep abrasions over his knees and
elbows. At casualty at the local hospital, the house officer examined the injuries and
noted the signs of acute inflammation. The wounds were cleaned with antiseptic
solutions. The patient was given a tetanus toxoid booster because he had received tetanus
toxin 10 years ago. The wound over the scalp was sutured and he was discharged.

OBJECTIVES:
1. List the various types of wound sustained by this patient.
2. Define inflammation and list the cardinal signs.
3. Describe the vascular and cellular events in the acute inflammatory response.
4. List the outcomes of acute inflammation.
5. Describe stepwise, the process of wound healing.
6. Outline various factors that influence wound healing.
7. List complications of poor wound healing.
8. List the various mild antiseptic solutions used in would cleansing.
9. Discuss the primary prevention of tetanus.
10. Discuss active and passive immunity.

January 7, 2008
GREAT IS 2019. WE PREVAIL.

PROBLEM 4

Transmission & Aetiology of Infection by Pyogenic Organisms

A two-year old girl was seen in the Pediatric Clinic with pustular lesions on her lower
limbs. Her mother, a graduate student, gave a history of the child being bitten by
mosquitoes. The doctor expressed concerns about organisms that are present on the skin
that may cause infection. The doctor also explained the transmission and course of
infection of pyogenic organisms. She took a sample of pus with a sterile swab and sent it
to the microbiology laboratory for Gram stain and culture. She also prescribed
antibiotics.

OBJECTIVES:
1. Discuss the aetiology of skin pustular lesions.
2. List the organisms that constitute the normal flora of the skin.
3. Discuss the transmission and pathogenesis of bacteria infection.
4. Describe the procedure of Gram stain.
5. Explain the significance of Gram stain results.
6. Outline the procedures for the culturing of bacteria.
7. Review the microbiology of staphylococci and streptococci including virulence
factors.
8. Describe the mammalian host defense mechanisms under natural barriers,
humoral mechanisms and cellular defense mechanisms.
9. Define the terms: antibiotic, bactericidal, bacteriostatic.
10. Classify antibiotics according to their mechanism of action.

January 7, 2008
GREAT IS 2019. WE PREVAIL.

MDSC 1101 Digestion and Metabolism

Problem 1
One Smart Cookie

One day while Sonny Smart was the intern on duty at the regional hospital, two female patients were
admitted to the nutrition ward for weight loss.

The severe weight loss suffered by Beatrice, a poor elderly woman, was occasioned by the fact that she had
very few teeth, and thus was unable to ensure proper diet. She admitted during history taking that for many
years she had neglected to brush her teeth, and when she did she used baking soda.

In the bed next to Beatrice was Kelly, a 21 year old aspiring model. Kelly was always overweight as a
child, and now as an adult, fed up of hearing what was good for her, she decided to embark upon a self-
administrated weight loss programme.

On admission Kelly appeared apathetic and unnaturally thin. Examination revealed bulging eyes, pallor
and a rapid pulse. Her tongue was smooth and inflamed and the corners of her mouth were
hypopigmented. She had dermatitis, brittle, easily pluckable hair, areas of petechiae and evidence of poor
wound healing. Her body mass index was 15.

Dr Smart invited the hospital nutritionist/dietitian to have consultations with both Beatrice and Kelly, and
they were slowly started on proper diets.

Objectives
1. Describe the gross anatomy of the buccal cavity with special reference to the tongue, its muscles,
innervations, blood supply, and lymphatic drainage.
2. Describe the embryology, histology, and gross anatomy of the salivary glands.
3. Describe the eruption of the teeth and discuss their functions.
4. Review the concept of recwaommended daily allowances (RDA) for energy and essential nutrients,
the derivation of RDA values, and their limitations.
5. List the constituents of a healthy or proper diet.
6. List the factors that determine energy and protein requirements in man.
7. Discuss the pathophysiology of Protein Energy Malnutrition.
8. Discuss the importance of mastication in digestion in humans and comment on the alteration of
nutrition in edentulous persons.
9. Briefly describe the principal eating disorders; anorexia nervosa, bulimina nervosa and binge eating
disorders.

January 7, 2008
GREAT IS 2019. WE PREVAIL.

Problem 2
Persistent Heartburn

A 65-year-old man with a six-month history of dysphagia, heartburn, anorexia, epigastric pain and
occasional vomiting was seen at the Medical Out Patient Clinic. At the clinic on questioning by his
doctor, he admitted taking antacid drugs in an attempt to relieve his heartburn, however, these were
not effective. He also gave a history of social alcohol use, irregular eating habits and smoking 5-10
cigarettes daily. A provisional diagnosis of peptic ulcer was made pending further investigation. The
doctor emphasized the need for lifestyle modification for efficient management of the patient.

Objectives
1. Describe the gross anatomy and histology of the oesophagus, stomach and duodenum.
2. Explain the mechanisms and control of deglutition.
3. Define oesophagitis, list the causes and the drugs used to treat oesophagitis.
4. Describe the physiology of gastric secretions and mention the functions of the main constituents
of gastric secretions.
5. Discuss the causes and consequences of altered levels of gastric acidity.
6. Define peptic ulcer and list aggravating factors.
7. Classify drugs used in the treatment of peptic ulcer, giving one example of each class.
8. Outline the components of a healthy diet and discuss the nutritional requirements of the elderly.
9. Is simply giving advice enough? Explain how health behaviour changes are essential for
facilitating dietary changes.

January 7, 2008
GREAT IS 2019. WE PREVAIL.

Problem 3
Weak William

William, a 17-year old male visited his GP complaining that he experienced early muscle
fatigue and a cramping muscle pain during strenuous exercise. He had observed that this
condition did not improve with additional exercise, with a sports drink containing
electrolytes, or with anti-inflammatory medications. His activities associated with daily
living are not affected. A blood sample taken after exercise contained very low level of
plasma lactate but plasma creatine kinase levels were elevated.

Objectives
1. Briefly explain the histology of the liver.
2. Describe the various sources of energy during short term and long term exercise.
3. Describe the pathways of glycogen breakdown and glycogen synthesis.
4. Explain the biochemical basis for glycogen storage disease.
5. Explain the central role of the liver in metabolism of nutrients.
6. Discuss the Cori cycle and its regulation.
7. Give an account of the nutritional needs and how they are met continuously from
birth to young adulthood.

January 7, 2008
GREAT IS 2019. WE PREVAIL.

Problem 4
A Tale of Two Gentlemen

While on call for the Medical Ward of the local hospital, final year student Eva admitted
two patients with liver problems. One was a 43-year old unmarried musician and the
other a 55-year-old businessman.

Mr. Yell OMellow, the musician had been touring the Caribbean for many years and had
fathered children in six different countries over the last 7 years. Recently he complained
of feeling weak, tired and unwell. He observed that he had a fever on and off, no
appetite, was nauseous and had dark urine. Additionally he noted some discomfort in the
right side of his abdomen. One of his friends also told him that his eyes looked yellow.

Upon examination of Mr. OMellow scleral icterus was confirmed. Eva noticed a tattoo
on his chest, an enlarged liver and tenderness over the right upper quadrant of his
abdomen in the area of his gall bladder. Results from blood analysis together with the
signs and symptoms suggested viral hepatitis.

Mr. C. Eeyo on the other hand was admitted because he was vomiting blood. He gave a
history of consuming more than 35 alcoholic drinks per week over several years.

On examination Mr. C. Eeyo was found to be agitated and somewhat disoriented. His
blood pressure was 90/40 mmHg, pulse 130 beats per minute and temperature 38oC. His
wbc and Hct were within normal limits. Mr. Eeyos abdomen appeared tense and
distended and his liver was palpable approximately 8 cms below the right costal margin.
His sclerae were also icteric and several spider angiomata were noted on the lower
extremities. After his blood analysis was completed an emergency endoscopy was
performed which revealed oesophageal varices. The initial treatment was with
intravenous vasopressin and nitroglycerin.

Eva thought about all the other things that could go wrong with the liver, and marveled at
this multi-tasked organ.

Objectives
1. Discuss the embryology, gross anatomy and histology of the liver, gall bladder and
biliary tract.
2. Briefly describe porto-caval anastomoses.
3. Explain the physiological bases of jaundice.
4. List the biochemical abnormalities that may result from hepatic dysfunction.

January 7, 2008
GREAT IS 2019. WE PREVAIL.

5. Discuss the relationship between the hepatic double circulation and liver function.
Relate the structure of the gall bladder to its function.significance and prognosis of
Hepatitis B & C viral infections.
6. Explain how some of the major causes of death and disability have a nutritional
component.

January 7, 2008
GREAT IS 2019. WE PREVAIL.

Problem 5
Hapless Harry

Several children between the ages of 5 years to 11 years old were brought to the health
centre all complaining of abdominal pain wtih several bouts of watery blood-stained
diarrhoea over the last 6-8 hours. One of them, 7 year old Harry, had fever and vomited
five times. On examination his temperture was 38.4C, skin turgor was markedly
diminished, his eyes appeared sunken and his tongue and lips were dry. His abdomen was
soft, non-tender, he had no guarding or rebound tenderness, and bowel sounds were
hyperactive. He was started on an intravenous infusion and blood was taken for CBC,
urea and electrolytes and stool tests were done. The DHV (District Health Visitor)
immediately informed the CMOH (County Medical Officer of Health) for the appropriate
public health actions.

Objectives
1. Describe the functional histology of the lower alimentary canal and associated glands.
2. Describe the changes in plasma electrolytes during chronic diarrhoea and their effects
on acid-base balance.
3. Discuss the role of the colon in water and electrolyte metabolism, and predict the
effect of any imbalances.
4. List the infectious agents associated with bloody diarrhoeal illness.
5. Define and list the differences between gastroenteritis, dysentery, diarrhoea and
enterocolitis.
6. Discuss the prevention and management of diarrhoea.
7. Explain the composition, importance, and route of administration for rehydration
therapy.
8. List the drugs used as antidiarrheal agents and briefly describe the mode of action of
diphenoxylate, loperamide, kaolin and pectin.
9. Identify ways in which foods can become contaminated.
10. Discuss the consequences of the lack of food hygiene and preventive control
measures.
11. Explain why Public Health is the responsibility of the whole society.
12. Explain how the science of Public Health is dependant on scientific knowledge of
causes of disease and how to prevent or treat, with systematic approach to identify
problems.

January 7, 2008
GREAT IS 2019. WE PREVAIL.

MDSC 1102 Cardiovascular and Renal

Problem 1

And the Beat goes on


A systolic murmur was detected in Joan when she was ten years old. However her mother
could not keep regular appointments with the doctor because of her inability to pay for
transportation.

When appointments were kept, the doctor recorded a progressive increase in the atrial
beat rate and a decrease in the ventricular beat rate over the course of the last ten years.
The doctor differentiated this murmur from the diastolic murmur of rheumatic fever.

Now at 19 years of age, Joans atrial beat was 78 bpm and the ventricular rate was 65
bpm. Her stroke volume obtained by echocardiography was larger than normal.

One day, when Joan was out shopping, she suddenly collapsed and was rushed to hospital
where she received an i.m. injection of quinidine. Unfortunately she died. The family was
distraught and was advised to seek counseling.

Objectives:
1. Describe the gross anatomy of the heart.
2. Describe the innervation and the conducting system of the heart.
3. Describe normal heart sounds.
4. Explain the physiological basis of heart murmurs.
5. Describe conduction of the cardiac impulse and the abnormalities of
conduction.
6. List drugs that are used in the treatment of cardiac arrhythmias.
7. Classify them according to Vaughn William Classification.
8. Discuss the epidemiology and impact of congenital lesions of the
cardiovascular system.
9. Discuss the familys response to loss as in the situation of death.
10. Discuss relevant issues in child health surveillance.

January 7, 2008
GREAT IS 2019. WE PREVAIL.

Problem 2
The Failing Heart

An executive medical examination revealed an enlargement of Mr. Deans heart.


Electrocardiogram showed Q waves in the inferior leads. Further investigations showed
abnormal cardiac enzymes. Mr. Dean explained to his doctor that in the past he had
suffered two heart attacks. The doctor made a diagnosis of heart failure arising from his
previous heart attacks. He then proceeded to discuss a treatment plan with the patient.

Objectives:
1. Describe the normal ECG tracing.
2. Give reasons for the abnormal Q waves seen in the inferior leads.
3. Describe the cardiac cycle with respect to pressure and volume changes.
4. Discuss the factors affecting normal cardiac output.
5. Using Frank Starling Law explain how the failing heart dilates to compensate
for loss of contractile power.
6. Identify drugs used to treat cardiac failure.
7. State (between digoxin and digitoxin) which of the cardiac glycosides is
preferred for treatment.
8. Explain the principal use of diuretics in treating patients with cardiac failure.
9. State the biochemical techniques used in the identification and measurement
of isoenzymes in blood and discuss the diagnostic value of these
measurements.

January 7, 2008
GREAT IS 2019. WE PREVAIL.

Problem 3

The Designer Heart


A team of engineers has contracted a medical intern to assist them in the computer
modeling of a heart and circulatory system. The model must depict the essential
anatomical, functional and physical characteristics of the heart and great vessels. The
engineers are specifically interested in obtaining details on how the heart is able to move
blood throughout the system of blood vessels. They are also curious about the
characteristics of blood which permit it to flow, the types of flow within the system and
the ability of the mechanisms to maintain unidirectional flow. The team also intended to
obtain critical vascular flow and resistance data and cardiac pressure and volume changes
from a simulation of an exercising heart.

Objectives:
1. Discuss the gross anatomy of the heart and great vessels.
2. Review the cardiac cycle with respect to pressure and volume changes.
3. Discuss factors that affect viscosity of blood and the physiological
consequences of these changes.
4. Explain the functions of the valves in the cardiovascular system.
5. Explain the determinants of resistance in the vessels and the relationship
between flow, pressure and resistance expressed as Poisseuilles Equation.
6. Explain the physiological changes caused by exercise.
7. Discuss laminar flow as it relates to the vascular system and the determinants
of Reynolds number.

January 7, 2008
GREAT IS 2019. WE PREVAIL.

Problem 4

Our Connected Systems


Dave is a stressed out CEO who has had uncontrolled hypertension (BP: 160/90) for the
past five (5) years. Lately he noticed puffiness around his face and ankles, and
experienced shortness of breath and fatigue. His blood pressure measurement was
190/115. His doctor had recently noticed abnormalities of his renal system. Dave has a
family history of hypertension and diabetes. Serum analysis which was requested
revealed the following results:

BUN: 80 mmol/L Na+: 148 mmol/L


Creatinine: 110 mmol/L K+: 4.8 mmol/L
Hb: 8.5 mg/dL 2+
Ca : 1.95 mmol/L

Urinalysis revealed proteinuria.

Objectives:
1. Describe the factors that regulate blood pressure in the short term.
2. Discuss the epidemiology of hypertensive disease including the role of
lifestyle and how it affects the prevalence of hypertensive disease.
3. Describe the pathophysiology of hypertensive disease.
4. Discuss the anatomy of the kidney.
5. Describe the histology of the filtration membrane.
6. List the functions of the kidney.
7. List the functional tests of the kidney.
8. Describe the factors that regulate blood pressure in the medium to long term.
9. Describe the role of the kidney in erythropoeisis.
10. Describe the role of the kidney in the aetiology of chronic anaemia.
11. Describe the role of the kidney in the regulation of calcium metabolism.
12. List the drugs used in the treatment of hypertension and describe the
mechanisms and sites of action.

January 7, 2008
GREAT IS 2019. WE PREVAIL.

Problem 5

Caroni versus Tobago


A family practitioner for over 20 years in County Caroni, Dr. Gene Diet observed that a
high proportion of his male patients in the age group 45-55 had suffered from Acute
Myocardial Infarction (AMI). Most of these patients were obese, had episodes of anginal
pain and biochemical investigations often revealed hyperlipidaemia. At the same time he
found that women of similar age had very few episodes of AMI but this trend changed in
the age group 55 and over where the incidence of AMI in women equaled that of men.

During the first week after moving to work at a Primary Health Care Centre in Tobago, a
medical student asked Dr Diet for help with a research project to study causes of
morbidity in the area. The doctor was surprised to learn that the leading cause or
morbidity in men aged 45-55 was hypertension.

Women over 55 showed a similar pattern of morbidity that the doctor saw in the Caroni
patients. Dr. Diet was fascinated by these ethnic, gender and age differences in the
morbidity patterns and decided to do a detailed study with the student on the
epidemiology of cardiovascular diseases in the two regions of the country. He considered
including in the discussion of the final report a section on managed health care plans
incorporating life style, diet and medication.

Objectives:
1. Discuss the blood supply and venous and lymphatic drainage of the heart.
2. Discuss the causes of ischemic heart disease and note the influence of diet,
age, gender, life-style (stress), habits (smoking), obesity and hypertension.
3. Discuss the biochemistry of plasma lipoproteins and the risks associated with
an abnormal lipid profile.
4. List the drug groups used in the treatment of hypercholesterolemia and
identify members of each group.
5. Discuss the incidence and prevalence of ischaemic heart disease and
hypertension in the Caribbean in relation to associated risk factors.
6. Describe the process (identifying tasks) used to undertake research.
7. Discuss the factors which influence what people eat (economic, social, ethnic,
religious, educational).

January 7, 2008
GREAT IS 2019. WE PREVAIL.

Problem 6

Surviving on Water
A 22-year old American marine was part of the military force sent to the Middle East
during the Gulf crisis. This was the first time he was posted to the sandy desert working
at temperatures around 49C (120oF).

Two days after his arrival, he was admitted to the sick room as he felt unusually tired and
fatigued. He complained of intense thirst and muscle cramps and had intermittent fainting
spells. He was infrequently passing small amounts of highly colored urine and
subsequently collapsed

The examining physician found his skin was hot and flushed, his eyeballs were soft and
sunken, and the rectal temperature was 41C (106F). He had rapid shallow breathing and
his blood pressure was 80/60 mmHg. He was diagnosed as having heat stroke. His
colleague was not affected as he regularly drank plenty of water to which a pinch of salt
was added.

Objectives:

1. Describe the gross anatomy and histology of the kidney.


2. Explain the mechanism of urine concentration.
3. Describe the transport processes in the kidney which are involved in the
maintenance of water and electrolyte balance.
4. Describe the different fluid compartments and distribution of electrolytes.
5. Discuss factors which regulate body water.
6. Explain the regulation of body temperature.
7. Discuss the effects of dehydration on cardiac and renal functions.

January 7, 2008
GREAT IS 2019. WE PREVAIL.

MDSC 2001 Respiration

PROBLEM 1: FACTORY WOES

Hilda Raucher is a 54-year old, 170 cm cement industry worker, who has worked in the kiln
section of the plant for the past 30 years. She complained of shortness of breath, which she
claimed was exacerbated by the often-dusty working environment. Upon questioning she
admitted to being a chain smoker and that she fatigues easily, even with light exertion. She had
developed chronic sinusitis and a persistent cough. On examination the doctor found generalised
hyper-resonance on percussion, prolonged expiratory phase in her breathing, bilateral expiratory
wheezes and coarse crackles over the right lower zone on auscultation and increased antero-
posterior chest diameter. Her nail beds were cyanotic and she had moderate pitting oedema of
the ankles. He concluded that the patients problems could be related to her occupation and
lifestyle and referred her to the pulmonary function laboratory for simple spirometry and blood gas
measurements. She was given appropriate treatment for the cough.

Her pulmonary function assessment and laboratory findings were as follows:

Pulmonary Function Tests Laboratory Values


Predicted vital capacity (PVC) 4.8 L Haemoglobin (g/dl) 14.5
Forced vital capacity (FVC) 3.6 L PaO2 (mm Hg) 48
Peak expiratory flow rate 300 l/min PaCO2 (mm Hg) 69
FEV 1.0 2.4 L O2 saturation (%) 78
HCO3 - 34

Based on the above data the pulmonologist recommended a full assessment to include
measurement of airway resistance (RAW ), lung compliance (CL), total lung capacity (TLC)
functional residual capacity (FRC) and residual volume (RV). The results are given in the table
below

Residual Volume (RV) 1.8 L


Functional Residual
3.0 L
capacity
Airway resistance RAW 3.2 cm H2O/l/sec
0.35 L/sec/cm
Lung compliance (CL)
H2O

OBJECTIVES:

1. Describe the anatomy of the nasal cavity, paranasal air sinuses and nasopharynx.
2. Draw a spirogram indicating lung volumes and capacities and factors affecting them.
3. Describe the physiological significance of the RV.
4. Discuss the physiological mechano-concepts involved in breathing and respiration (movement
of air into and out of the lung; RAW ; forces generated by the respiratory muscles to move the
air, CL, surface tension and the defense system of the respiratory system).
5. Compare the values of the patient with normal values.
6. Describe the common symptoms and signs associated with respiratory disease.
7. Discuss the pharmacology of drugs used in the treatment of cough (suppressants,
expectorants and mucolytics).
8. Discuss the pharmacology of:
drugs used as bronchodilators (salbutamol, theophylline, adrenaline and ipratropium)
drugs used as prophylactic treatment of asthma (sodium cromoglycate) and
glucocorticoids.

January 7, 2008
GREAT IS 2019. WE PREVAIL.

January 7, 2008
GREAT IS 2019. WE PREVAIL.

PROBLEM 2: MORE THAN A BLOCKED BRONCHUS


Mr. Sevrin Restrictor, a 61-year-old retired welder consulted his physician because he was
experiencing shortness of breath during exertion and was coughing up blood (haemoptysis) for
three weeks. He had recently experienced mild chest pain in the upper part of his thorax on the
right side and noticed a remarkable hoarseness of his voice.

Examination revealed diminished breath sounds in the upper lobe of the right lung and fine rales,
particularly at the base of the left lung. An obscuration of the hilum of the right lung by a mass
(suggestive of a tumor) was detected on the chest radiograph, in addition to fine diffuse mottling
of the left lung. A bronchoscopy and mediastinoscopy with a biopsy were ordered for this patient.

During endoscopy of the bronchial tree, a growth obstructing the origin of the right superior lobar
bronchus was observed and the biopsy material obtained from this bronchus and the right
tracheobronchial lymph nodes showed the presence of malignant tumor cells.
The patient was referred to the pulmonary function laboratory for evaluation.

Pulmonary function studies showed the following:

Indices Change
Total lung capacity
Functional residual capacity
Residual volume
FEV1/FVC
Lung compliance
DLCO
PaO2 (at rest) 75 mm Hg
PaCO2 (at rest) 36 mm Hg
% Saturation (at rest) 97%
PaO2 (during exercise) 62 mm Hg
PaCO2 (during exercise) 35 mm Hg
% Saturation (during
exercise) 90%

OBJECTIVES:

1. Describe the anatomy of the lower respiratory tract and the right and left lungs (To include
arterial supply, venous and lymphatic drainage of the lungs, bronchial tree and innervation).
2. Discuss the aetiopathology of the restrictive pulmonary disease.
3. Distinguish between obstructive and restrictive diseases and describe the various laboratory
methods used to assess ventilatory function.
4. Discuss factors affecting the diffusing capacity of the lungs
5. Outline the major immune defenses in the upper and lower respiratory tract.

January 7, 2008
GREAT IS 2019. WE PREVAIL.

PROBLEM 3: THE PUMP IS DOWN

Ms. Venus Obesino is a 39 year old unmarried doctor, who weighs 110kg. She is 150 cm tall. She

has tried most of the slimming drugs without much success. She recently stumbled upon the fact

that some 50 years ago 2, 4 dinitrophenol was given as a drug to help patients lose weight. She

self administered the drug, with near fatal consequences.

On recovery she learnt that the difference between weight loss and death is only a small
concentration change in dinitrophenol, making the drug dangerous. She agreed to do further
studies on blockers of the electron transport chain, their sites of action and to analyse the effect
of the blockers on the chemiosmotic theory.

OBJECTIVES:

1. List the components of the electron transport chain, and describe their organisation and
function.
2. Explain the term uncoupler and list the common inhibitors and uncouplers of the electron
transport chain, indicating their site of action.
3. Discuss oxidative phosphorylation with emphasis on the chemiosmotic theory.

January 7, 2008
GREAT IS 2019. WE PREVAIL.

PROBLEM 4: CREEPING DISTRESS

A 23-year old, semiconscious man is brought into the emergency room following an automobile
accident. He is tachypnoeic and cyanotic. Physical examination reveals a small laceration and
flailing on the right lower anterolateral thoracic wall. There appears to be no air entering or
leaving the wound and therefore it is assumed that the pleura has not been penetrated.

The patient is placed on positive pressure endotracheal respiration, which corrected the cyanosis
and abnormal chest wall movement.

Radiological examination confirms fractures of the 4 th through 8th ribs in the right anterior axillary
line and of the 4th through 6th ribs at the right costochondral junction. There was no evidence that
bony fragments had penetrated the lungs or of a pneumothorax. The laceration was sutured and
the chest bound in bandages, with the endotracheal ventilation maintained.

Several hours later the right side of the thorax was found to be more expanded than the left,
moved less during respiration and the cyanosis returned. A chest radiograph showed a horizontal
level of fluid in the right pleural cavity, which was interpreted as haemothorax. A ventilation-
perfusion scan was done and showed marked unevenness of ventilation.

A puncture of the thoracic wall was performed and a negative pressure drain (chest tube) was
inserted into the right pleural space.

OBJECTIVES:

1. Describe the gross anatomy of the thorax, pleurae and pleural cavities and the muscles of
respiration.
2. Explain the concept of ventilation/perfusion ratio with reference to the regional differences in
ventilation and perfusion of the lungs.
3. Define and quantitate dead space (VD), alveolar ventilation (VA) and discuss the factors
affecting them.
4. Define hypoxaemia and hypoventilation, and list the major causes of hypoxaemia.
5. Discuss the pathophysiology and implications of:
6. Cyanosis
7. Hyperventilation
8. Hypoxaemia
9. Haemothorax

January 7, 2008
GREAT IS 2019. WE PREVAIL.

PROBLEM 5: ALPS HYPE

Don Fit decided to celebrate his 21st birthday by joining a mountain climbing expedition to the
Swiss Alps. Don is in excellent condition; he runs 5 miles daily and plays squash and soccer on a
regular basis. Don underwent a complete medical examination, including blood gas
measurements before the climb. His PaO2 was 100 mm Hg and all other parameters were within
the range of normal. The physician recommended that he take acetazolamide. The expedition
was to Mont Blanc, which is 18,000 feet above sea level. The barometric pressure at this level
is 415 mm Hg.

Don found that during the ascent to 10,000 ft he felt tired and developed symptoms of
tachypnoea, laboured breathing, tachycardia, dizziness, nausea and fatigue. The experienced
team leader encouraged him to breathe from a tank of 100% O 2 and enquired whether he had
taken acetazolamide before the climb. The rest of the climb was uneventful.

Don on his return home visited the doctor to determine whether any alteration had occurred in his
blood profile.

OBJECTIVES:

1. Review the anatomy of the pulmonary circulation.


2. Review hypoxia, and the various types of hypoxia.
3. Discuss the transport of oxygen and carbon dioxide.
4. Explain how the exchange of oxygen and carbon dioxide is regulated.
5. Discuss the oxyhaemoglobin curve and its response to changes in pH, oxygen, and 2, 3
bisphosphoglycerate concentrations.
6. Discuss the physiological changes, which occur at high altitude.
7. Discuss the role of the peripheral and central chemoreceptors in the control of respiration.
8. Discuss the role of acetazolamide with respect to acid-base balance.

January 7, 2008
GREAT IS 2019. WE PREVAIL.

MDSC 2002 Neurosciences and Behaviour


PROBLEM 1

Born With a Lump

A male infant was born with a large midline mass in the lumbar region of the back that
was covered with a membranous sac. Within a few days the mass grew larger and
ulcerated leading to fluid leakage from the mass. The infant was taken to the hospital
where the attending physician detected bilateral neurological deficits in the lower limbs.
An MRI also revealed partial herniation of the medulla oblongata and the cerebellum
through the foramen magnum. The doctor informed the mother that her child was
suffering from a congenital abnormality.

Objectives

1. Describe the formation, circulation, drainage, constituents and functions of


cerebro-spinal fluid (CSF).
2. Describe the anatomical structures in the mass found in the lumbar region.
3. List the common congenital malformations of the brain and spinal cord.
4. Discuss the basis for the neurological deficits demonstrated by the doctor and the
structures implicated in the deficits.
5. Describe Arnold-Chiari malformation and the sequence of events associated with
this malformation.

January 7, 2008
GREAT IS 2019. WE PREVAIL.

PROBLEM 2

Back Stabbing For Real

James was rushed to the hospital after he was stabbed in the back. The examining doctor
noted that below James navel, he did not respond to pain and temperature on his left
side, or proprioception on his right side. James was referred to a neurosurgeon for
appropriate management.

Objectives

1. Describe ascending (sensory) neuronal tracts in the spinal cord and brain.
2. Describe the segmental innervations of the body and dermatomes.
3. Determine the level of spinal cord lesion based on the presenting neurological
symptoms and signs.
4. Relate segments of the spinal cord to vertebral levels.
5. Discuss the possibility of recovery of sensation.

January 7, 2008
GREAT IS 2019. WE PREVAIL.

PROBLEM 3

To Transmit Or Not To Transmit?

Epilepsy is a syndrome that arises when the excitatory neurotransmitter, glutamate,


stimulates neurons to start firing wildly and uncontrollably, sometimes resulting in
seizures or loss of consciousness. In a recent study, a team of scientists induced epilepsy
in rats and found to their great surprise that there was an increase in abnormal glial cells
in the cerebral cortex. One of the roles of normal glial cells is to mop up excess glutamate
but the abnormal glial cells produced increased amounts of glutamate.

Current treatments of epilepsy are aimed at slowing down brain function to reduce the
neuronal firing, but these have unwelcome side effects, such as drowsiness due to their
action on the reticular activating system. Researchers used the rat models to analyse three
GABAmimetic drugs commonly used to treat epilepsy. They found that in addition to
slowing neural firing, the drugs reduced the type of chemical signaling that causes
astrocytes to release glutamate. This may contribute to why the drugs work, the
scientists suggested.

Objectives

1. List the types of glial cells in the central nervous system and their functions;
indicate which one of them is affected in the scenario above.
2. Classify glutamate receptors.
3. Briefly describe the general structure and mechanisms of action of ionotropic and
metabotropic receptors.
4. List the types and causes of epilepsy in humans and discuss their underlying
mechanisms.
5. Discuss the role of GABAmimetic drugs in the treatment of epilepsy.
6. Discuss the management of epilepsy.
7. Discuss the role of the Reticular Activating System in consciousness.

January 7, 2008
GREAT IS 2019. WE PREVAIL.

PROBLEM 4

Proteins Slow the Progress of Parkinson's Disease

Researchers claim to have slowed the progress of Parkinsons disease in rats by injecting
two proteins into a part of the brain that has been implicated in Parkinsons disease. The
proteins prevented the brain cell loss associated with the disease.

Until now, most therapies have concentrated on replenishing the neurotransmitter, but
that does not stop the brain cell loss and so have had limited success. Now, working in
rats, a team of scientists have succeeded in halting brain cell death in the vital brain
region. The researchers used a genetically engineered virus to transfer the genes for the
two proteins Sonic Hedgehog and Gli-1 to the brain cells of a group of adult rats
referred to as Group I. One week after the proteins had been injected, a drug that induces
Parkinsons disease was given to the rats, as well as to a second group of rats that had not
received the proteins referred to as Group 2.

Four weeks later, the Group 2 rats had lost 80% of neurons in the vital brain region.
Whereas the Group I rats had only suffered 20% - 30% brain cell loss in the same region.

The results of this study were submitted to the journal Brain Research for publication.
Reviews were favorable and though one referee suggested that a third group of rats be
analysed to improve the reliability of the results.

Objectives

1. Name the region of the brain in which the scientists injected the proteins.
2. Draw a diagram to illustrate the direct & indirect pathways in the basal ganglia;
indicate the neurotransmitters involved in the pathways.
3. Describe the synthesis of the neurotransmitter that mammalian brain cells
manufacture to prevent progression of Parkinsons disease; indicate the rate-
limiting enzyme.
4. Describe all the receptors for the transmitter (identified in 3) and possible second
& third messenger systems for the transmitter.
5. Discuss the use of genetically engineered virus vectors for gene transfer.
6. Discuss the role of the third groups of rats in improving the reliability of the
research.

January 7, 2008
GREAT IS 2019. WE PREVAIL.

PROBLEM 5

Memory Loss in Mice & Humans

A group of scientists discovered that injecting fragments of the protein beta-amyloid from
human patients with Alzheimers dementia into the brains of a group of mice caused the
animals to forget chores they had just been taught. When further electrophysiological
experiments were done, both long-term potentiation and long-term depression were found
to be compromised.

Objectives

1. Describe the gross anatomy, histology and embryology of the human cerebral
cortex.
2. List the pathological features that would be expected to develop in the brain of the
mice.
3. Describe the common classifications of learning.
4. Describe the common classifications of memory.
5. List neurotransmitters and brain regions associated with memory and learning.
6. Compare (and contrast) long-term potentiation and long-term depression.

January 7, 2008
GREAT IS 2019. WE PREVAIL.

PROBLEM 6

Upper or Lower????

Mr. Jones, a 63 year old Afro-Trinidadian man awakes one morning with weakness on
his right side. He also complains of difficulty seeing and has slurred speech. He is rushed
to the accident and emergency department where he is examined. Findings include a
history of hypertension, right-sided hemiparesis (equally in the arm and leg) with
diminished pinprick and two point discrimination of his right side. His muscle tone on the
right is reduced and so are his tendon reflexes.

Six months later when Mr. Jones is seen in the out-patient clinic he is walking with a
limp and has little functionality of his right arm. However tendon reflexes on his right
side are now increased and he has a positive Babinski sign and increased muscle tone on
the right side.

Objectives

1. List the components of the neurological examination and briefly describe how
they are assessed.
2. Describe the arterial supply to the brain indicating which artery/arteries could
have been blocked to cause the stroke in the above patients. (The use of a diagram
is recommended).
3. Describe the ascending pathways for pain and two-point discrimination.
4. Describe the primary descending motor pathways.
5. Explain the physiological basis for: (i) a simple stretch reflex (ii) the primitive
reflexes.
6. Compare and contrast upper and lower motor neuron lesions.

January 7, 2008
GREAT IS 2019. WE PREVAIL.

PROBLEM 7

Hearing Loss Is More Than The Ears

Mrs. Johnson, a 70 year old retired music teacher, has been experiencing a gradual loss in
hearing over the last 10 years. Even with her hearing aid turned to full volume, she can't
make sense of speech, especially when two or more persons speak at the same time
during a conversation.

Audiometric testing showed sensory-neural deafness and a PET scan revealed that there
was less than normal activity in the midbrain. The scan also showed that fewer nerve
fibers infiltrated the midbrain and that there were lower levels of calcium entering and
leaving the nerve cells.

The specialist explained, much to Mrs. Johnson's disappointment, that a cochlear implant
was unlikely to improve her hearing. He further advised Mrs. Johnson's daughter that her
mother would benefit from persons speaking more slowly to her and engaging her in a
polite conversation where people take turns.

Objectives

1. List the common causes of hearing loss in the different age groups.
2. Describe the anatomy of the ear.
3. Describe transduction of sound in inner ear.
4. Describe the auditory pathways.
5. Describe the processing of hearing & speech in the CNS.
6. Describe how polite conversation improves Mrs. Johnsons understanding of
speech.
7. Describe the various tests for hearing.
8. Explain how hearing aids & cochlear implants work.

January 7, 2008
GREAT IS 2019. WE PREVAIL.

MDSC 2101 Endocrine and Reproduction

Problem 1 HYPER- AND HYPO-ACTIVITY

Kenny was small for his age. Although he was 22 years old he looked like a boy of 11.
He was not very active and was mentally challenged. Kennys doctor had previously
indicated that Kenny was a thyroid dwarf.

Kennys 30-year-old tutor, who had developed normally, also appeared to have a thyroid
problem. She had just visited her physician and informed him that she was losing weight
although she was eating incessantly. She complained of muscle weakness, and fatigue
during exercise. She had also developed irregular menstrual periods, and her skin was
often warm and moist.

A medical examination by her doctor revealed that her thyroid gland was enlarged.
Laboratory tests indicated high levels of total serum T4, free T4 and low levels of serum
TSH.

OBJECTIVES:

1. Describe the embryology, anatomy and histology of the anterior pituitary.


2. Describe the embryology, anatomy and histology of the thyroid gland.
3. Describe the synthesis, storage and release (including regulation) of thyroid
hormones.
4. Describe the biological actions (both anabolic and catabolic) of the thyroid
hormones.
5. Explain the laboratory findings of the 30-year-old tutor.
6. Describe the causes and effects of abnormally high concentrations of thyroid
hormones.
7. Describe the causes and effects of low concentrations of thyroid hormones.

January 7, 2008
GREAT IS 2019. WE PREVAIL.

Problem 2 - TOO MUCH STEROID

Anne, a 60-year-old woman who had suffered from chronic arthritis for years, had been
treated over the past year with 50 mg/day of prednisolone, which she had been obtaining
over the counter at the local drug store. She had observed recently that her skin had
become quite thin and easily bruised, and her best friend teased her about her moon face.
Her arms and legs always felt extremely weak, and an X-ray revealed osteoporosis.

One day while crossing the pavement she slipped and fell and sustained a fracture of the
femur and after admission to hospital she was scheduled for surgery. Blood tests
revealed elevated levels of glucose. Anne omitted to tell the doctor that she had been
taking prednisolone and the medication was not given in hospital after admission. During
the operation, which was done three days later, her blood pressure suddenly fell to 80/50
mm Hg and the doctors had great difficulty maintaining the blood pressure. Post-
operatively, the doctors discovered that Anne had been taking a steroid prior to her
admission and they suggested that this could have affected her adrenal gland function.

OBJECTIVES:

1. Describe the embryology, gross anatomy, and histology of the adrenal gland.
2. Describe the physiological actions of glucocorticoids.
3. Describe the pharmacological actions of glucocorticoids.
4. Discuss the aetiology and pathophysiology of hypersecretion of glucocorticoids.
5. Discuss the tests used to assess adrenal cortical function.
6. Discuss the complications of steroid use.
7. Discuss the pathophysiology of adrenal failure.

January 7, 2008
GREAT IS 2019. WE PREVAIL.

Problem 3 - DIABETES ANYONE

A group of third year medical students decided to undertake a study of their classmates
for their Community Health class project, to determine the number with the potential for
developing non-insulin-dependent diabetes mellitus (NIDDM, Type II). The group first
administered a questionnaire to the class members to ascertain baseline data, including a
family history. Blood samples were taken for determination of several parameters. Since
one member of the group had a graduate degree in Biochemistry, he decided to do further
analyses on the insulin receptor binding activity. Arising from the success of the project,
the group was awarded a grant to extend the study to a community in the North West
Regional Health District. The group had to include in the study a component on health
education to assist the public in understanding how blood sugar is normally controlled,
and aspects of diabetes prevention and management.

OBJECTIVES:

1. Describe the embryology, gross anatomy and histology of the endocrine pancreas.
2. Describe the pathophysiology of the different types of diabetes mellitus.
3. Discuss methods used in the diagnosis of diabetes mellitus.
4. Describe hormonal control of blood glucose levels in patients with NIDDM.
5. Describe strategies for the prevention and management of Type II diabetes.
6. Classify and describe the hypoglycemic drugs available for the treatment of Type
II diabetes.
7. Classify insulin preparations as short, medium and long acting and identify
sources of insulin preparations.

January 7, 2008
GREAT IS 2019. WE PREVAIL.

Problem 4 - TRYING TO FILL THE VOID

A fifty-two year old male lawyer whose first marriage had remained childless after ten
years was recently re-married to a young woman who was anxious to start a family. He
noticed very early in his marriage that he was having difficulty with micturition and had
to get up frequently at night to void his bladder. He consulted his doctor, who, on
examination of his prostate gland, found it to be enlarged and tender. The doctor also
requested a semen sample for analysis as well as blood samples for serum FSH/LH and
androgen estimation. The lawyer sought advice on the potential influence of his current
problem on his past and future fertility, as well as his libido and ability to have erections.

OBJECTIVES:

1. Describe the embryology, gross anatomy and histology of the male reproductive
system.
2. Discuss the process of spermatogenesis and the maturation and storage of
spermatozoa.
3. Discuss the physiology of erections.
4. Discuss the physiology of ejaculations.
5. Define the composition and functions of seminal fluid.
6. Discuss the causes of male infertility and reduced libido.
7. Describe the parameters used in evaluating production and quality of semen.

January 7, 2008
GREAT IS 2019. WE PREVAIL.

Problem 5 AMENORRHEA AND/OR PREGNANCY

Jane, a 25-year-old woman, who had been having her normal 28-day menstrual cycle
since the age of 13 years, presented with a history of amenorrhea for three months. A
urine test ruled out the possibility of a pregnancy. She also reported that her mother had
died in an automobile accident four months ago. The young woman wondered which of
her ovarian functions were disrupted, and how the disruptions affected the rest of her
reproductive system. The doctor ordered laboratory tests for the estimation of serum
concentrations of FSH, LH, prolactin and the ovarian sex steroids before any medication
could be contemplated.

Evita, a twenty-one year old actress had also missed her last three periods. She had
gained some weight and noticed slight tenderness in her breasts. Her doctor performed a
urine pregnancy test and diagnosed that she was pregnant. She wondered about the
processes that had occurred in her body following intercourse, as well as, the changes that
were taking place in her. Since it was her first pregnancy, her doctor advised her about
regular antenatal care as an adjunct for an uneventful parturition. She was also given
literature about lactation and the merits of breast-feeding.

OBJECTIVES:

1. Describe the embryology, gross anatomy and histology of the female reproductive
system including the placenta.
2. Discuss the causes of amenorrhea.
3. Identify the major physical and hormonal changes associated with pregnancy.
4. Outline the principles underlying tests used in the diagnosis of pregnancy.
5. Discuss the hormones involved in breast development and lactation.
6. Discuss the merits of breast feeding.

January 7, 2008
GREAT IS 2019. WE PREVAIL.

Problem 6 THE ALPHA AND OMEGA OF FERTILITY

Brigit was turning into a well-developed thirteen year old girl who was somewhat
embarrassed by the initiation of menses and her breast development, which was
becoming a source of attention from some of the young boys in the neighbourhood. Her
own attitude to the opposite sex was beginning to shift noticeably. Some of the boys
were also just beginning to manifest growth spurts, facial hair growth and a deepening of
their voices. Brigit wondered what was causing the changes in her body, as well as, the
changes in the young boys.

In contrast, Brigit's 53-year-old grandmother was experiencing irregular menses,


excessive sweating at night and frequent mood swings. Her doctor discussed the pros
and cons of hormone replacement therapy with her.

OBJECTIVES:

1. Discuss the factors underlying the onset of puberty in males and females and
menopause in females.
2. Describe the physiology (including regulation) of puberty in males and females.
3. Discuss the physiology and management of climacteric changes.
4. Classify oestrogens (natural and synthetic) and recognize their use in hormone
replacement therapy.
5. Discuss the controversy surrounding hormone replacement therapy.
6. Describe the psychological factors associated with the onset of puberty or
menopause.

January 7, 2008
GREAT IS 2019. WE PREVAIL.

MDSC 2102 Muscles, Bones and Joints

Problem 1
A Lemon A Day

Devin was a member of a crew which was going to re-enact Columbuss journey across
the Atlantic in a replica of the Santa Maria. As he was about to board the ship, his wife
Gail-Ann joked that she hoped he had packed some lemons because she wanted him to
come back with all his teeth. He knew she was referring to a disease which used to afflict
sailors on long voyages, causing their skin to become spotted and their gums to bleed.
He remembered reading one description which referred to the sailors losing all their
strength and not being able to stand on their feet. He later asked the ships cook if they
had lemons on board. The cook replied that they didnt have lemons but they had plenty
of other fresh fruit on board.

Devin realised that he did not know why lemon juice prevented the disease or if other
fruit would do the same job. He thought of his cousin Navin who had complained about
the difficulties in getting fresh fruit for his family (his job had moved him to a remote
area in the Arctic) and wondered what the effects would be on Navins children Stacey
and Simone, whose bones were rather soft and flexible. He also was not sure if a cod
liver oil supplement to the childrens diet could compensate for a deficiency of fresh fruit
and avoid problems with their health.

OBJECTIVES:
1. Describe the histology of supportive connective tissues (bone, cartilage) with
special reference to their matrix.
2. Describe the structure and function of collagen and the role of ascorbic acid in the
synthesis of hydroxyproline.
3. Describe the development of bone. Contrast types of ossification. Discuss the
role of epiphyseal plates in growing bones and factors controlling bone growth
and ossification.
4. Discuss the role of vitamin D in the process of ossification of bones and outline
the mechanism of calcium regulation.
5. Classify various types of continuous joints (fibrous, cartilaginous, osseous) and
specify their locations.
6. Describe the pathophysiology and treatment of scurvy.
7. Describe the pathophysiology and treatment of rickets and osteomalacia.

January 7, 2008
GREAT IS 2019. WE PREVAIL.

Problem 2
The Young Scientist

John a curious second-year medical student was performing a series of experiments upon
a nerve-muscle (skeletal) preparation. He noted that when he stimulated the nerve once,
the muscle would twitch. But if he increased the frequency of stimulation, a point would
be reached when the muscle would stop twitching and remain contracted. John wondered
if this was what occurred during muscle 'cramps' and whether it was in any way related to
a reduction in muscle ATP with repeated stimulation. He also was not sure about the
differences between the skeletal muscle fibres of different colour and wanted to test
whether the same thing would occur in smooth muscle because he vaguely remembered
someone telling him that smooth muscles cannot undergo tetanus.

In another series of experiments John observed that if he added acetylcholine to the bath,
without stimulating the muscle, the preparation would contract. This even occurred when
he sectioned the nerve. However contraction was greatly reduced if the concentration of
calcium in the bath was lower than normal.

OBJECTIVES:
1. Describe the histology and ultrastructure of skeletal muscle and contrast muscle
fibre types, their distribution and innervation. Describe the functional histology of
the neuromuscular junction.
2. Describe the mechanism of skeletal muscle contraction and functional roles of
slow and fast twitch muscle fibres. Explain the role of acetylcholine and calcium
in muscle contraction.
3. Describe a muscle twitch. Explain how contraction occurs in skeletal and smooth
muscles and note their differences.
4. Define the term tetanus and explain how it comes about.
5. Discuss the different sources of fuel for skeletal (fast and slow twitch fibres) and
smooth muscles.
6. Discuss the effects of oxygen debt and ATP depletion on muscle function.
7. Discuss the possible biochemical basis and pathophysiology of muscle cramps
and their management.

January 7, 2008
GREAT IS 2019. WE PREVAIL.

Problem 3
A Carnival Dancer

Mr. Green, a 37-year-old Trinidadian taxi driver was complaining to his family doctor
about lower back pain, which began just after the last carnival. On questioning he told
the doctor that he and his friends had a lot of fun participating in recent carnival
activities. Mr. Greens dancing apparently involved excessive movements of his trunk,
which led to his lower back problem.

On examination of his back, the doctor noticed moderate tenderness of the lower
paravertebral muscles and movements of the lumbar spine were painful and limited.
Radiographic study revealed a spina bifida occulta of the 1st sacral vertebra, while an
isotope scan of the lumbar vertebrae was normal. A muscle relaxant was prescribed.

Mr. Greens father often suffered with lower backbone problems because he had a
slipping lumbar vertebra and Mr. Green wondered if he could have a similar problem
with his own vertebra, which might lead to his present condition.

OBJECTIVES:
1. Describe the gross anatomy and embryology of the vertebral column with special
emphasis on its articulations, mobility and stability.
2. Discuss common congenital abnormalities of the vertebral column, their
manifestations and complications.
3. Describe the gross anatomy of the muscles of the back and movements they
produce.
4. Discuss the effect of extreme movements on the vertebral column.
5. Contrast the factors which can cause lower back pain.
6. Discuss the diagnostic tools for backbone problems.
7. Name groups of common muscle relaxants and describe the use of
benzodiazepines to provide muscle relaxation.

January 7, 2008
GREAT IS 2019. WE PREVAIL.

Problem 4
Spoilt Concert

Ms. T., a beautiful middle-aged famous Trinidadian singer, was invited to New York in
January to give a concert. On the way from the airport to her hotel she could not avoid
the blast of the chilling wind on her face and neck. The next morning she looked in the
mirror and was shocked because her face looked crooked. While she was able to move
her head and neck freely, she could not close her left eye; and on drinking, water dribbled
from the left corner of her mouth.

She recalled a similar problem in the past when the dentist, treating her teeth, blocked a
nerve in her mouth by an anaesthetic. At that time all symptoms disappeared on the same
day. However, on this occasion the specialist to whom she was referred, explained after
examination of her face and radiographs of her skull that she could have a nerve
compression in the small opening of the skull also known as Bells palsy and her new
condition would require a more prolonged period of treatment. Since her concert was
scheduled for the same evening, the singer had to cancel it.

OBJECTIVES:
1. Describe the gross anatomy of the skull and the individual cranial bones.
2. Describe the gross anatomy of the muscles of the head and neck and explain their
actions.
3. Explain the procedure of inferior alveolar nerve block and anatomical reasons for
the possibility of facial nerve block during that procedure.
4. Discuss the pathophysiology of facial paralysis (Bells palsy), its manifestation
and diagnosis.
5. Discuss a negative effect of facial paralysis on other organs of the head.

January 7, 2008
GREAT IS 2019. WE PREVAIL.

Problem 5
Clicos Marathon & Unexpected Problems

Mr. Thomson, a young marathon competitor from Jamaica was preparing himself for
international and regional races. In order to improve his speed during the last part of the
race he began weight training. While training one day he noticed that straining to lift
heavy weight could be very tiring and rapidly produce muscle fatigue even though his
arm muscles were not moving.

He arrived in Trinidad to take part in a marathon competition, sponsored by CLICO, and


on his way from Piarco airport was involved in a car accident in which he sustained an
injury of his right shoulder with traumatic dislocation of the humeral head, closed
fracture of the surgical neck (confirmed on radiographs) and deep lacerated wound in the
right posterior triangle of the neck. He was treated in the department of surgery at Mount
Hope. His blood sample was sent to clinical laboratory for calcium and alkaline
phosphate determination. After repair, the lacerated wound and bone fracture were healed
within a normal period of time due to proper deposition of collagen.

However, six months after the surgical correction the injured glenohumeral joint still had
significant limitation of movement which was associated with postoperative scars and
deficiency of the synovial fluid and its components in the cavity of this joint.

OBJECTIVES:
1. Describe the gross anatomy of the glenohumeral joint and the muscles acting on
it.
2. Identify potential areas of weakness of the glenohumeral joint and the structures
that maintain its stability.
3. Describe the muscular triangles of the neck and their contents.
4. Review the functions of collagen including its role in the process of wound
healing.
5. List the different types of glycosaminoglycans with their composition, site of
occurrence and functions.
6. Explain the clinical importance of serum calcium and alkaline phosphatase levels.
7. Discuss the length-tension relationship and other biophysical relationships in
muscles.
8. Briefly describe the physiological changes the body undergoes during exercise.

January 7, 2008
GREAT IS 2019. WE PREVAIL.

Problem 6
Unhappy Triad

Mr. F., a popular 22-year-old forward of the National Soccer Team of Trinidad and
Tobago has missed several matches due to a right knee problem, known as a runners
knee. In the ensuing games he was very successful and contributed many goals toward
the teams victory. At his last game he was tackled by the defender while running and
dribbling the ball. The defenders shoulder hit his left knee from the side when his foot
was firmly planted on the ground. He felt a severe pain and fell. As a result of left knee
injury he was taken by ambulance to the Port of Spain General Hospital where Unhappy
triad was diagnosed. Recovery after surgical treatment was prolonged and complicated,
resulting in residual instability of the knee joint, associated with improper locking and
unlocking mechanisms. His teammates wondered if he would ever be able to play with
the team again.

OBJECTIVES:
1. Describe the gross anatomy of the knee joint and the muscles acting on it.
2. Explain the locking and unlocking mechanism of the knee joint.
3. Discuss common knee injuries.
4. Describe the pathophysiology of chondromalacia patellae (runners knee) and its
management.
5. Define the term: Unhappy triad. Describe its etiopathogenesis, diagnosis,
management, complications.
6. Discuss factors leading to the instability of the knee joint.

January 7, 2008
GREAT IS 2019. WE PREVAIL.

Problem 7
Defective Gene

Johnny, a 5-year-old boy, was brought to the doctor by his mother. She was extremely
worried and very concerned about the way Johnny walked and his generalized muscular
weakness. She explained to the doctor that his condition had been getting progressively
worse during the past three years.

The doctor observed an exaggeration of the lumbar lordosis when Johnny was standing,
and a waddling gait when he walked. Almost all his muscle groups were poorly
developed with exception of his calf muscles, which looked excessively large. He also
observed that Johnny had a tendency to stand and walk on the front of his feet with his
heels off the ground. Apart from muscle hypertrophy of legs, the ankle joints and bones
forming them did not have any lesions.

Further enquiries by the doctor revealed that Johnnys older brother did not have any
health problems, but his cousin had a congenital anomaly known as a clubfoot.

Tests of individual Johnnys muscles and muscle groups demonstrated a consistent


pattern of weakness. Blood test revealed an increased level (50 times) of a serum creatine
kinase (SCK). Electromyography showed low electrical activity of the tested muscles. A
muscle biopsy was ordered for analysis of the gene and its protein, dystrophin.

OBJECTIVES:
1. Describe the gross anatomy of the ankle joint and the muscles of the leg acting on
it.
2. Describe the development of the lower limbs and their common congenital
anomalies.
3. Understand the principles behind electromyography and how it can be used to aid
the clinical diagnosis of muscular disorders.
4. Discuss the aetio-pathogenesis of Duchennes muscular dystrophy (DMD), its
clinical manifestations and diagnosis.
5. Describe the structure, the location, the attachment of dystrophin to the
cytoskeleton of the muscle fibre and its role in muscular dystrophy.
6. Explain the reason for the high level of the creatine kinase in a blood serum in
patients with DMD.
7. Discuss the mode of inheritance of DMD.

January 7, 2008
GREAT IS 2019. WE PREVAIL.

MDSC 3310 Pathology and Microbiology/Pharmacology/Public Health and


Primary Care

.1. Chest Pain

An obese, dyslipidaemic, hypertensive male complained of a six-month history of a


vague precordial aching sensation which extended into his left shoulder and down the
inside of his left arm. It occurred particularly at times of exertion and occasionally at
night. However, he did not seek medical attention until the pain became more severe,
persistent and was accompanied by shortness of breath. When seen in Casualty, the
patient appeared to be anxious and restless with cool diaphoretic skin. His pulse was
regular and somewhat thready, at 98/minute. Heart sounds were distant. BP 180/100
mm.Hg, Resp. rate 24/min. On examination of the chest, inspiratory rales were heard at
the lung bases. An electrocardiogram obtained in casualty showed deep Q waves with ST
segment elevation. Blood was collected for biochemical investigations. A diagnosis of
acute myocardial infarction was made.

OBJECTIVES:

.1. Discuss the epidemiological pattern of ischaemic heart disease and its primary,
secondary and tertiary prevention.
.2. List the differences between angina pectoris and myocardial infarction including
the meaning of the terms stable and unstable angina.
.3. Discuss the pathogenesis of acute myocardial infarction including predisposing
factors.
.4. Outline the time dependent gross and microscopic findings seen in acute
myocardial infarction.
.5. List the complications of acute myocardial infarction and explain the pathogenesis
of each.
.6. Discuss the lipid profile of a patient with heart disease.
.7. List the biochemical markers used to diagnose myocardial infarction and explain
the time dependent relationship of biochemical markers to myocardial infarction.
.8. Discuss the management of the patient with an acute myocardial infarction and
explain the rationale for all drugs used.
.9. In a patient who has had a myocardial infarction, describe the use of thrombolytic
and anti-platelet agents.
.10. List the drugs used for the treatment of angina and describe their mechanisms of
action and side effects.

Tutors Guidelines:
Students should be able to discuss the use of thrombolytic drugs and anti-platelets

drugs in acute myocardial infarction based on their pharmacological mechanism of

action.

January 7, 2008
GREAT IS 2019. WE PREVAIL.

Obj. 3 - These predisposing factors should include age, male sex, obesity, smoking,

low HDL and high LDL.

.2.

January 7, 2008
GREAT IS 2019. WE PREVAIL.

.3. Pneumonia

A 27-year-old man presented to the emergency room complaining of fever and cough.
Two days previously, he noted the onset of weakness and malaise, which forced him to
go to bed earlier than usual. The next morning he had a shaking chill that lasted 15
minutes. One hour later his temperature was 39.4C. Several hours later he noted the
onset of right lower chest pain, which was aggravated by taking a deep breath and by
coughing. That evening his cough became productive of brown-coloured sputum. All the
symptoms had progressed by the time he arrived at the emergency room.

Physical examination revealed an acutely ill young man with BP 120/80, pulse rate
120/minute, respiratory rate of 32/minute and temperature of 39.4C. His lips and nail
beds were cyanotic. There was increased vocal fremitus, dullness and rales over the right
lateral chest.

Gram stain of the sputum showed many polymorphonuclear leukocytes and gram-
positive diplococci. Chest x-ray showed consolidation of right lower lobe. Twenty-four
hours after the administration of appropriate antimicrobial therapy, the patient became
afebrile. He eventually made a complete recovery.

OBJECTIVES:

.1. Discuss the etiology and classification of pneumonia.


.2. Discuss the structures and mechanisms present in the respiratory system that
protect against infection.
.3. What populations are at risk for pneumonia and what preventative measures are
available?
.4. Describe the pathogenesis of pneumonia.
.5. Describe the gross and microscopic features of the classic stages of evolution of
lobar pneumonia, bronchopneumonia and primary atypical pneumonia.
.6. Describe the complications of pneumonia.
.7. Describe the haematological findings in a case of pneumonia.
.8. Outline the biochemical test that constitutes blood gases.
.9. Describe acid-base balance during hypoventilation and hyperventilation.
.10. Discuss respiratory acidosis and alkalosis.
.11. Discuss the role of the plain chest radiograph in the diagnosis of pneumonia and
other chest infections.
.12. Describe the major side effects of penicillins and describe the diagnosis and
treatment of hypersensitivity reactions.
.13. Discuss microbial resistance to the -lactam antibiotics.
.14. Identify -lactamase resistant penicillins (cloxacillin, nafcillin, methicillin) and -
lactamase inhibitors (Sulbactam, clavulanic acid).

.4. A Case of Joint Pain

January 7, 2008
GREAT IS 2019. WE PREVAIL.

A 30-year-old female complained of painful wrists and palms for the past 3 months. She
also admitted to her physician that she was experiencing stiffness in her hands on
mornings, and that her pain was exacerbated when she sewed. She was an upholsterer and
her ability to do her job was being compromised.
On examination, the wrists and metacarpophalangeal joints of both hands were swollen
and tender. There were no deformities, nodules or vasculitic lesions. Blood was taken for
measurements of CBC, C-reactive protein, rheumatoid factor, ANF and dsDNA.

She was diagnosed as possible autoimmune disease and was prescribed non-steroidal
anti-inflammatory drugs. This medication proved only partially effective and after two
months her knees and ankles were similarly affected, so she was referred to a
rheumatologist.

OBJECTIVES:

.1. Classify common joint disorders.


.2. Define autoimmunity and the factors predisposing to the development of
autoimmune disease.
.3. Discuss the immunological tests used to diagnose and monitor rheumatoid
arthritis (RA) and systemic lupus erythematosus (SLE).
.4. Discuss the gross and microscopic findings in RA.
.5. Describe the microscopic examination of the joint aspirate.
.6. Discuss the complications of chronic RA.
.7. Describe the anaemia of chronic disorders.
.8. Describe the mechanism of action of COX 1 and COX 2 NSAIDs.
.9. Describe the pharmacology and adverse effects of disease-modifying anti-
rheumatic drugs used in the management of severe RA.
.5.

January 7, 2008
GREAT IS 2019. WE PREVAIL.

.6. Prison Infections

A 66-year-old prisoner was sent to the doctor for persistent cough and difficulty in
breathing. A few weeks earlier he was alarmed when bright red blood came up in one of
his coughing bouts. He had been in prison for the past year and lately was not keeping
well, had lost some weight, suffered spells of diarrhoea for no apparent reason and was
generally weak.

On physical examination, his BP was 130/80, pulse rate was 110/minute, respiratory rate
was 26/minute and his temperature was 38C. Movement of the chest wall was slightly
reduced on the right side. His chest radiograph showed a lesion suggestive of a cavity in
the right infra-clavicular area. He had a positive tuberculin skin test. Serological tests
proved that he was positive for HIV antibodies.

OBJECTIVES:

.1. Contrast the morphological features of primary and secondary (reactivation)


tuberculosis.
.2. Describe the laboratory diagnosis of mycobacterial infection and common
opportunistic infections.
.3. Discuss the role of the BCG vaccine in the prevention of tuberculosis.
.4. Describe the administration and interpretation of a Mantoux test.
.5. What are the first and second line drugs used in the treatment of tuberculosis?
Discuss their side effects.
.6. Describe the natural history of HIV infection and strategies for prevention of HIV
infection.
.7. Discuss the tests used to diagnose and monitor HIV infection.
.8. Discuss the pathogenesis and morphological features of pneumocystis pneumonia.
.9. Classify drugs used in the treatment of HIV and explain their mechanism of
action.
.10. Describe the pharmacokinetic properties and toxic effects of the antiretroviral
drugs.
.11. Labelling and stigma are important barriers in the doctor-patient relationship.
Discuss with respect to ill prisoners.
.12. With regard to the Caribbean Charter for Health Promotion, discuss what would
be useful policies for persons living with HIV/AIDS.

Tutors note: Enlarge the discussion of HIV prevention to include mother-to-child


transmission.
Obj. 12 Students should discuss public policies that would enable the patient to be
respected at home, in the workplace and in health care institutions.

January 7, 2008
GREAT IS 2019. WE PREVAIL.

Problem 5. Lymphadenopathy

A 30-year-old female who has not yet started a family, presented to her GP complaining
of painless, persistent swellings in the neck. She did not admit to fever, night sweats or
weight loss and she had no risk behaviour for HIV.

On examination, the only significant finding was bilateral 4cm nodes in the lower
cervical region; there was no hepatosplenomegaly. Her BP was 110/70, and her weight
50kg.

On investigation her chest x-ray was normal. Mantoux test, HIV and VDRL screens were
negative. Lymph node biopsy showed nodular sclerosing Hodgkins disease, and she
subsequently had a whole body CT scan that confirmed the presence of cervical
lymphadenopathy only.

She was counselled about her diagnosis (Stage IIA Hodgkins lymphoma) and treatment:
combination chemotherapy with chlorambucil, vincristine, procarbazine and
prednisolone. She was concerned about prospects of having children after treatment with
these drugs.

After four courses of this treatment, she had gained 10kg and her BP was 140/100 with a
fasting blood glucose of 11.1mmol/l.

OBJECTIVES:

1. Discuss the causes of lymphadenopathy in children and adults.


2. Make a checklist of questions in history taking which are relevant to the causes
you
identify.
3. Summarize the laboratory investigations needed to evaluate the patient with
lymphadenopathy.
4. Describe the histological subtypes of Hodgkins lymphoma.
5. Define staging of tumours. Describe the staging system for Hodgkins lymphoma.
6. Describe the mechanisms of action and side effects of drugs used in the treatment
of
Hodgkins lymphoma.
7. Identify the impact of the diagnosis on the patients life.
8. What is the role of the family physician in assisting this patient?

Tutors guidelines: Broadly classify lymphomas into non-Hodgkins and Hodgkins


lymphomas.

January 7, 2008
GREAT IS 2019. WE PREVAIL.

Problem 6. Sickle Cell Disease/Chronic Renal Failure

PW is a 52-year-old mother of three with sickle cell disease. She has attended clinic since
childhood. She first became aware of her disorder when she missed sitting her end-of-
year test in standard three. The previous weekend the family had gone to Maracas beach
and after she came out of the water, she felt a bit cold but was having too good a time to
complain.
The following day she had generalized body pains that did not respond to paracetamol
syrup. Two days later she developed a cough and fever. Her next memory of the event
was being in hospital attached to a needle and a bag of fluid. Many doctors were around
the bed and she remembered hearing the words chest infection.

Since that first admission, she has had numerous hospital admissions for chest problems
and has been on prolonged bed rest for ulcers on the right ankle.

She now has three grown up children, all of whom are well. She was transfused during
her last pregnancy. Her daughter in the USA is pregnant and would like to know if her
baby will have sickle cell disease.

Results of blood tests done at her last clinic visit:


Hb4.8g/dl MCV 73.5fl, reticulocyte count 11%, WBC 15.8 x 109/l,
platelets 315 x 109/l, urea 35mg/dl, creatinine 3.3mg/l, K+ 5.1mmol/L.
Total bilirubin 6.05mg/dl indirect 4.53mg/dl direct 1.52mg/dl

Her blood pressure was BP 140/90.


She is presently on erythropoietin and folic acid and has been given a chronic renal
failure diet by the dietitian.

OBJECTIVES:

1. Arrange the problems encountered by persons with sickle cell disease on an age
chart from birth to the 6th decade. Include the psychosocial problem that may be
prevalent at each range.
2. Discuss the chest infections to which the sickle cell disease patient is susceptible.
3. What might you expect to find in a sickle cell patient who died of acute chest
syndrome?
4. Create a checklist of the complications of blood transfusion under the following
headings.
immunologic
microbiological
immediate
delayed
5. List the effects of sickle cell disease on the spleen along with the immunological
and clinical sequences.
6. List the common underlying causes of systemic embolization and infarction.
7. Describe the patho-physiological sequence of events that most commonly result in
pulmonary infarction.

January 7, 2008
GREAT IS 2019. WE PREVAIL.

8. Review the physiologic functions of the kidney and the role of erythropoietin in
the management of chronic renal failure.
9. Explain the types of anaemia expected as a result of deficiency of folic acid and
erythropoietin.
10. Describe a chronic renal failure diet.
11. What support systems are available to help the chronically ill patient to cope?

Tutors guidelines: Make reference to the Society of Inherited Medical Blood


Disorders.

January 7, 2008
GREAT IS 2019. WE PREVAIL.

Problem 7. Acute Lymphoblastic Leukaemia

LJ, an 8-year-old boy, presented with a four-week history of fatigue and pallor, a one-
week history of back pain and a one-day history of a low-grade fever (38C). Physical
examination revealed multiple small, mobile, non-tender, bilateral cervical, axillary and
inguinal lymph nodes. Laboratory tests indicated Haemoglobin of 9.9 g/dl, a platelet
count of 125 x 109/l and a WBC count of 14 x 109/l.
Further investigations led to a diagnosis of acute lymphoblastic leukaemia (ALL).

He was pre-treated with allopurinol, which continued during his 28-day course of
induction chemotherapy, which consisted of the following:

Drug Treatment Days

vincristine 1, 8, 15, 22
prednisone 1-28

L-asparaginase 4, 6, 11, 13, 15, 18, 20, 22


daunorubicin 1, 8, 15, 22

In addition, methotrexate was administered intrathecally on days 15 and 29. Radiotherapy


to the central nervous system was administered as part of the CNS prophylaxis.

Upon completion of induction therapy, bone marrow examination demonstrated only


normal marrow cells, consistent with remission of leukaemia. Although he was in
remission, consolidation therapy was initiated with cyclophosphamide, 6-thioguanine and
cytosine arabinoside. Intrathecal methotrexate was again included in the regimen.

OBJECTIVES:

1. Describe the epidemiological features of leukaemia, including environmental and


occupational risk factors.
2. Evaluate the parameters on the blood count commenting on their values with
respect to the normal ranges.
3. What further investigations were needed to come to the diagnosis of ALL?
4. What are the clinical complications of ALL?
5. Discuss the difference between induction and consolidation chemotherapy.
6. Explain the mechanism of actions of the antineoplastic drugs used.
7. What is the purpose for pre-treating with allopurinol?
8. Why is methotrexate given intrathecally?
9. What are some of the problems that the child and his caregivers experience in
living with this diagnosis?
10. What are the roles of the primary care physician in prevention and screening for
cancer?
11. What resources are available to help in the management of patients with
leukaemia?

January 7, 2008
GREAT IS 2019. WE PREVAIL.

Problem 8. Autoimmune Haemolytic Anaemia/Systemic Lupus Erythematosus

Mavis is a 40-year-old East Indian who at age 30 noticed sudden onset of tiredness
associated with yellow eyes.
On questioning, she gave a history of pains in multiple joints and extreme tiredness. The
significant findings on examination were icterus, pallor and a palpable spleen 6cm below
the left costal margin.

Her CBC showed the following:


Hb 5.0 g/dL MCV 102fl WBC 6 x 109/L Platelets 145 x 109/L

She was referred from her GP to haematology clinic.


A blood film showed raised reticulocytes and spherocytes. Her direct Coombs test was
positive. Further investigations showed her to have a positive ANF. Urine showed
moderate proteinuria and hematuria.

OBJECTIVES:

1. List the causes of jaundice under the headings:


pre-hepatic
hepatic
post-hepatic
2. Use a systems approach to identify the important autoimmune diseases that affect
the various organ systems.
3. Classify autoimmune haemolytic anaemias.
4. Outline the importance of the direct and indirect Coombs test in the evaluation of
haemolytic states.
5. Review the causes of macrocytic anaemia and the relevant findings on blood film
examination.
6. Describe the clinical manifestations of SLE.
7. Describe the laboratory diagnosis of SLE.
8. Describe the morphological changes in the kidney in a patient with SLE.

January 7, 2008
GREAT IS 2019. WE PREVAIL.

Problem 9. Intracranial Bleed

A 50-year-old salesman was admitted to the Priority Care Facility. He was accompanied
by his wife who said he had complained of a severe headache that developed while
straining at stool about 10 hours before. This was followed by a sudden loss of
consciousness. She also told the doctor that her husband had suffered with high blood
pressure for the past 12 years and had been taking his anti-hypertensive drugs on and
off. She could not remember the last time he had visited the family physician to check
his blood pressure.

On examination, the patient was drowsy, confused and breathing heavily. There was neck
stiffness. BP = 170/110 mm Hg, Pulse 80/min. Temperature was normal and apart from
the elevated blood pressure the CVS exam showed an enlarged heart. Bilateral retinal
haemorrhages were seen on ophthalmoscopic examination. A lumbar puncture was
performed in PCF and the CSF showed xanthochromia. A CT scan was then ordered and
revealed subarachnoid haemorrhage in the basal cisterns. A ruptured aneurysm was
suspected. Laboratory investigations: Hb = 13.5g/dl, WBC = 10 x 109/l, platelets = 200 x
109/l, PT and PTT normal.

The patient was admitted to the ward and treatment started with analgesics and anti-
hypertensives including a calcium channel antagonist. A referral was sent to the
neurosurgeon but the patients condition quickly deteriorated and he became comatose
before he could be seen. He died shortly afterward. Autopsy confirmed a ruptured berry
aneurysm in the Circle of Willis. Bleeding from the aneurysm extended in the subjacent
cerebral cortex and brain stem. The heart and kidney showed pathological changes
consistent with long-standing hypertension.

OBJECTIVES:

1. Define aneurysm and discuss the morphological classification of aneurysms.


2. Discuss the pathogenesis of berry aneurysms and describe the morphological
features of berry aneurysms.
3. List the common ages and sites of occurrence of berry aneurysms and discuss
the clinical significance.
4. Discuss the laboratory investigations of a patient with suspected intracranial
haemorrhage and hypertension.
5. Describe the morphological features of hypertensive cerebrovascular disease
including intracerebral haemorrhage.
6. Describe the morphological changes in the heart and kidneys in hypertension.
7. Describe the circulation of CSF.
8. Discuss normal and abnormal constituents of CSF and the conditions
associated with them.
9. List causes of xanthochromia.
10. List precautions taken when doing a lumbar puncture.
11. Discuss the medico-legal implications of sudden death.

January 7, 2008
GREAT IS 2019. WE PREVAIL.

Problem 10. Dysphagia

For the past four years Git, a 56-year-old businessman of 152cm height and weighing
98kg was under self-medication with antacids for water brash and heartburn. A year ago
he had been to a GP because the symptoms persisted and he was experiencing
odynophagia on drinking grapefruit juice. He was prescribed Zantac and advised by the
doctor about stress management related to his busy lifestyle.

Presently, he complains of dysphagia and feels food is sticking in his chest. The GP
referred him to a gastroenterologist who advised him to undergo gastroesophagoscopy.
On enquiry he was found to be a smoker and admits to heavy daily consumption of
alcohol.

A CBC was done and revealed a decreased Hb with a microcytic picture on the blood
film.
The following findings were noted upon endoscopy:
a) Barretts changes in the lower third of the oesophagus with ulceration.
b) Nodular growth at the oesophago-gastric junction.

A biopsy was taken from the growth for histological evaluation.


Upon receipt of the histopathology report, treatment protocol was discussed with the
patient.

OBJECTIVES:

1. Describe the gross and microscopic anatomy of oesophagus and the gastro-
oesophageal junction.
2. List the causes of heartburn and dysphagia.
3. Discuss the aetio-pathogenesis of gastro oesophageal reflux disease and describe
the morphology, complications and management of reflux esophagitis.
4. Describe the morphological changes associated with Barretts oesophagus.
5. Describe the epidemiology, risk factors, aetio pathogenesis, morphology, spread,
staging and management of oesophageal cancer.
6. List and describe some common tests for gastrointestinal function test e.g.
Schilling test, D-xylose absorption test, Breath test for H. pylori.
7. Explain the significance of stool examinations.
8. Describe the test for faecal occult blood.
9. Review possible causes of microcytic anaemia.
10. Describe the process of smoking cessation.
11. Discuss the primary care approach to the patient with dysphagia.
12. Discuss the drugs used in the treatment of GORD. (e.g. H2 antagonists,
cimetidine, proton-pump inhibitors e.g.omeprazole, sucralfate, antacids).

Tutors guidelines: Briefly discuss obesity and its relevance to this case.

January 7, 2008
GREAT IS 2019. WE PREVAIL.

Problem 11. Acetaminophen Toxicity

RM, a 36-year-old man with a history of alcohol dependence was brought by his brother
to the emergency room. He had gone to visit RM and found him drunk. A bottle of
acetaminophen (500mg) was under the table. Apparently, RM was taking these tablets
for headache for several days and wasnt eating properly. His brother was very worried
since he noticed RMs skin was yellow and that RM was very distraught and suffering
from general malaise. He had also been taking sedatives at bedtime for several months to
help him sleep.

A blood sample was taken for analysis. On examination, RM appeared ill with tenderness
in the right upper quadrant of abdomen and jaundice. While he was being examined he
vomited blood. Blood test results were as follows: Prothrombin time 15 seconds-control
11 seconds, aspartate aminotransferase 2000 U/L, alanine aminotransferase 1200 U/L,
total serum bilirubin 5.5mg/dL.

He was transferred to the intensive care unit where adequate management was provided
and therapy with N-acetylcysteine was initiated. After a week, signs and symptoms
began to resolve and laboratory tests started returning to normal values. RM was now
willing to attend a rehabilitation centre to quit alcohol since he was told he was digging
his own grave.

OBJECTIVES:

1. Describe the mechanism of action of acetaminophen and describe the various


eliminative pathways for acetaminophen in humans.
2. Describe the clinical manifestations and mechanism of hepatotoxicity caused
acetaminophen.
3. What potential risk factors could have increased this individuals susceptibility to
acetaminophen hepatotoxicity?
4. List useful agents to treat acetaminophen hepatotoxicity and discuss their
mechanism of action and effectiveness.
5. Discuss the medico-legal responsibilities required for prescription drugs.
6. What are the risk factors for alcoholism?
7. What social support structures can be used during rehabilitation?
8. Discuss the importance of liver function tests in this patient.
9. Discuss the tests to differentiate alcoholic liver cirrhosis from obstructive liver
disorder.
10. Describe the morphology of liver in alcoholic liver disease.

Tutors guidelines: Students need to know how to define a unit of alcohol for

different alcoholic drinks and the upper limit of consumption for men and women.

January 7, 2008
GREAT IS 2019. WE PREVAIL.

Problem 12. The Cascadura Hunter

A 30-year-old man presented to the Couva Health Centre with a four-day history of fever,
yellow eyes, headache, myalgia and nausea. He had been swimming in the Caroni River
in an attempt to catch cascadura 12 and 5 days previously. He had always been fit and
well.

On examination, he was haemodynamically stable, pyrexial (39.1C), and had


meningism, cervical lymphadenopathy and abdominal tenderness. An urgent CBC
showed neutrophils 9.29 x 109/l, platelets 102 x 109/l, but no other abnormality. He was
admitted to hospital where cerebrospinal fluid analysis was normal. He was started on
cefotaxime 2g four times daily, and oral doxycycline 200mg, once daily to cover
meningococcal, streptococcal, gram negative and atypical infections. Further blood
investigations revealed urea 8.2mmol/L, creatinine 155mmol/L, aspartate
aminotransferase 139 IU/L, alanine aminotransferase 90L, gamma glutamyl transferase
99L, platelets 68 x 109/l and a prothrombin time of 14.4 seconds, control 10 seconds.

The patient became increasingly breathless and hypoxaemic. A chest radiograph showed
cardiomegaly and ECG widespread ST segment elevation. He was started on a
dobutamine infusion plus 40% oxygen administered by facemask. The ST segment
changes subsequently resolved and on completion of a weeks course of antibiotics he
was discharged.

OBJECTIVES:

1. Discuss the epidemiology of Leptospirosis in the Caribbean.


2. Review the mode of transmission, incubation period, diagnosis and management
of Leptospirosis.
3. List the complications of Leptospirosis.
4. Describe the control and prevention of Leptospirosis.
5. List the tests that evaluate liver function.
6. Explain bilirubin metabolism.
7. Discuss the abnormal LFT in leptospirosis.
8. Discuss renal function tests.
9. Explain how you would estimate GFR.
10. Discuss the pharmacology of dobutamine.
11. Explain the use of cefotaxime and doxycycline.

Tutors guidelines: Students should discuss the pharmacology of dobutamine vis a vis
dopamine.

January 7, 2008
GREAT IS 2019. WE PREVAIL.

Problem 13. Glomerular Disease/Urine Analysis

A 9-year-old previously healthy boy presented with puffiness of face and swelling of
extremities of four days duration. His mother mentioned that he recovered from the
Virus a few weeks ago. On physical examination, he was afebrile. Heart rate = 92/min.,
RR = 22/min. and BP = 100/60 mm Hg. He had 3+ pitting oedema of feet. Abdomen was
soft and normal on palpitation. Urinalysis revealed 4+ proteins and had no rbcs or pus
cells. Blood results were as follows: Serum albumin - 1.8g/dL

He was treated with prednisolone for six weeks and re-evaluation showed proteinuria and
BP was 130/90 mm Hg. His parents were advised that he should have a renal biopsy.

OBJECTIVES:

1. List the organ systems associated with peripheral oedema.


2. Describe the diagnostic criteria for nephrotic syndrome.
3. Compare nephritic syndrome with nephrotic syndrome.
4. List the causes of nephrotic syndrome.
5. Describe the aetio-pathogenesis and pathology of nephrotic syndrome in
childhood.
6. Outline the indications and contraindications for renal biopsy.
7. Discuss the complications of nephrotic syndrome.
8. Describe the management and prognosis of this patient.
9. Explain the procedure for doing a 24-hour urine collection and list the error
sources.
10. Explain urine analysis under the following headings: urine microscopy, dipstick
analysis.
11. Explain the sources of interference when doing a dipstick analysis.

Tutors guidelines:
Diagnostic criteria include urine, blood and clinical findings.

Nephrotic Syndrome to describe morphology of kidney in each of the common

causes listed above (Obj.5).

January 7, 2008
GREAT IS 2019. WE PREVAIL.

Problem 14. Urinary Tract Infection, Thrombocytopenia, Prostate Cancer

GA is a 59-year-old company director, who presented to his GP with a three-day history


of painful urination and a one-day history of rigors and fever. A digital rectal
examination revealed an enlarged nodular prostate. He had been taking ranitidine for
heartburn.

The GP made a diagnosis of acute urinary tract infection and prostatic enlargement for
further evaluation and prescribed cotrimoxazole empirically. When he returned one week
later for review, he noticed bruising on the arms, legs and inside the mouth. Blood was
taken for relevant investigations. The results were:
Hb 15g/dl, WBC 6.5 x 109/l, platelets 3 x 109/l.

He was admitted to hospital and the haematologist, who was called, ordered the
following investigations: ANF, PSA, bone marrow aspirate.

The ANF was normal, the PSA was elevated and the marrow showed plentiful
megakaryocytes with normal erythropoiesis and myelopoiesis.

A diagnosis of drug-induced, immune mediated peripheral destruction of platelets was


made. This was thought to be due to cotrimoxazole. The patient was treated with
intravenous immunoglobulin and steroids. One week later his results were:
Hb 14.5g/dl, WBC 16.6 x 109/l, platelets 331 x 109/l

Steroids were tailed off. He had a needle biopsy of the prostate two weeks later and was
diagnosed as having prostatic adenocarcinoma, Gleason score 6.

OBJECTIVES:

1. Review the structure of the normal bone marrow.


2. Identify the causes of low platelets under the headings:
1. Production failure
2. Increased consumption/utilization
3. Peripheral destruction.
3. Review the mechanisms of immune mediated hypersensitivity disorders (Gell and
Coombs classification).
4. Utilizing this classification, identify the mechanism by which a drug may cause
thrombocytopenia, and by which I-V immunoglobulin moderates the disease.
5. What is the reason that corticosteroids are tailed off rather than stopped?
6. Discuss the factors that pre-dispose to urinary tract infections.
7. Discuss the therapeutic treatment and management of a patient with UTI.
8. What are the advantages and disadvantages of empirical prescribing of
antibiotics?
9. Discuss the risk factors and epidemiology of cancer of the prostate.
10. Discuss the role of the PSA in the diagnosis and treatment of prostate cancer.
11. Describe the morphology and grading of carcinoma of the prostate.
12. Discuss the staging of prostate cancer.

January 7, 2008
GREAT IS 2019. WE PREVAIL.

January 7, 2008
GREAT IS 2019. WE PREVAIL.

Problem 15. Ovarian Mass

A 34-year-old female, who was seen by her physician for unrelated complaints, was
found on routine pelvic examination to have a palpable left ovarian mass. Past medical
history and family history were unremarkable except that her mother had died of breast
cancer. The remainder of the physical examination was unremarkable, as was the routine
haematological work-up. Tumour markers including HCG, and a-fetoprotein were
negative but cancer antigen 125 (CA 125) was moderately elevated. The patient was
explored with a midline abdominal incision and a 5cm cystic mass was found in the left
ovary. A frozen section was interpreted as serous cystadenocarcinoma, and a small
volume of ascitic fluid was interpreted as positive for malignant cells. The tumour was
staged surgically as FIGO* stage IC. A hysterectomy with bilateral salpingo-
oophorectomy was performed. Postoperatively, the patient was treated with adjuvant
chemotherapy consisting of paclitaxel, and cisplatin.

OBJECTIVES:

1. Outline a pathological classification of ovarian neoplasms, indicating the age


group most frequently affected, the relative frequency, and the malignant potential
of each type.
2. Discuss the predisposing factors and pathogenesis of ovarian neoplasms,
including related cytogenetic abnormalities.
3. Discuss the importance of frozen sections in histopathology.
4. Which ovarian tumours are most likely to be hormonally active?
5. List some common tumour markers.
6. Discuss the classes of common tumour markers.
7. Discuss the usefulness of tumour markers including CA125 in diagnosis,
prognosis, treatment, and screening.
8. Discuss the treatment of hormone-dependent cancers in women.
9. List specific toxicities of paclitaxel and cisplatin.
10. Classify cancer chemotherapeutic agents based on their mode of action and
describe the major toxicities for drugs in each class.

*International Federation of Gynaecologists & Obstetricians

January 7, 2008
GREAT IS 2019. WE PREVAIL.

Problem 16. Breast Lump

A 50-year-old accounts clerk visited her family physician after feeling a small, irregular,
painless lump in her right breast. She had been on hormone replacement therapy for two
years. On questioning she revealed that her mother had died of breast cancer at age 60
and that she was worried about her increased risk. She had never had a mammogram. On
examination, the doctor found a firm to hard, irregular, mobile mass in the lower outer
quadrant of the breast. He referred her to a general surgeon who performed a fine needle
aspirate of the lump in his office and told her to return in two days for the result. When
she returned, the surgeon reported that the pathologists report described the presence of
malignant cells in the aspirate. An open biopsy of the lump was scheduled a week later.

A diagnosis of infiltrating duct carcinoma-Grade 1 was made on histology. The tumour


was 2.0cm in diameter. Foci of duct carcinoma in-situ were seen adjacent to the invasive
tumour. In addition, there was fibrocystic change and sclerosing adenosis with duct
epithelosis in the surrounding breast. The tumour was estrogen receptor positive and
HER-2 negative. After extensive discussion with the breast surgeon, a modified radical
mastectomy with axillary node clearance was performed. No residual tumour was found
in the breast and 12 axillary lymph nodes were negative for metastatic disease.

The patient was treated with Tamoxifen postoperatively and 18 months later is currently
free of disease.

OBJECTIVES:

1. Discuss the factors that increase a womans risk of developing breast cancer.
2. Discuss the current recommendations for the screening of breast cancer.
3. Describe the gross and microscopic features of benign breast diseases including
proliferative and non-proliferative fibrocystic change.
4. Explain the relationship between the different kinds of benign breast disease and
the subsequent risk of developing carcinoma.
5. Explain what is meant by carcinoma in-situ and describe the recognised types that
occur in the breast.
6. Describe the morphology of infiltrating lobular carcinoma and the four common
subtypes of infiltrating duct carcinoma.
7. Explain how primary tumour size, lymph node status, oestrogen and progesterone
receptor status, HER-2 receptor status and histological grade help predict survival
(or assist management) in breast cancer.
8. Describe the staging of breast cancer.
9. Discuss CA27-29 and CA15-3 with respect to the diagnosis and management of
breast carcinoma.
10. Discuss the advantages and disadvantages of hormone replacement therapy
(HRT).
11. List drugs used in the treatment of breast cancer relating to cytotoxic
chemotherapy and hormone receptor positive drugs.
12. Explain the different mechanisms of action of the hormonal therapies of
tamoxifen and anastrozole.

January 7, 2008
GREAT IS 2019. WE PREVAIL.

13. Discuss the role of Herceptin (including its major toxicity) as a monoclonal
antibody in treating breast cancer.
Tutors guidelines:
Classify biochemical tumour markers giving examples of these. Please stress the

various types of tumour markers.

January 7, 2008
GREAT IS 2019. WE PREVAIL.

Problem 17. Cervical Tumour

Ms. V. E. a 43-year-old, visited her local health centre where the nurse asked her whether
she had ever had a Pap smear. She replied that she had never had one and after a brief
explanation by the nurse, she agreed to have one done that day, mainly because she had
recently noted a bloodstained vaginal discharge. On further enquiry, she had four children
by three different partners and all pregnancies had been uncomplicated.

The nurse took the Pap smear in the clinic and told her to return in three weeks for the
result.
Exfoliated cervical cells were also sent for assessment of HPV status. On her return, Ms.
V.E. was advised by the medical officer that her Pap smear report showed abnormal cells.
She was then given a referral letter to the Gynaecology Out-patients Clinic at the Mt.
Hope Womens Hospital for further investigation. Some weeks later, a colposcopic loop
biopsy of her cervix was taken and sent to the histopathology laboratory.

The histology report showed invasive squamous carcinoma of the cervix, which extended
to the margin of the biopsy specimen. A radical hysterectomy was later performed and
her tumour was staged IIB.

1. Discuss the epidemiology of and risk factors for cancer of the cervix.
2. Discuss the pathogenesis of cervical cancer as it relates to known risk factors.
3. Discuss the role of the immune system in protection against oncogenic viruses
such as HPV.
4. Outline the principle of one method of detecting viral nuclei acids in cervical
cells.
5. Describe the pre-cancerous squamous lesions of the cervix using the standard
classifications of these lesions.
6. Discuss efficacy of the Pap smear as a screening test for cervical pre-cancer and
outline the requirements for taking a satisfactory smear.
7. Outline the reporting systems used for cervical smears.
8. Review the criteria for appraisal of a screening program.
9. Describe the gross and microscopic features of the common forms of invasive
cervical cancer.
10. Describe the staging of cervical cancer.
11. Discuss the basic principles of chemotherapy in cancer treatment.
12. Discuss the most appropriate antineoplastic agents for the management of this
cervical cancer patient.
13. Discuss the role of combined chemotherapy.

Tutors guidelines: Students should summarise criteria for appraising practicability,


efficacy and appropriateness of screening programs using Wilsons criteria.

January 7, 2008
GREAT IS 2019. WE PREVAIL.

Problem 18. Diabetes mellitus (1)

A 48-year-old diabetic woman is hospitalised because of an ear infection. Three months


previously she noticed the onset of redness, swelling, and drainage from her left ear. She
had been treated by her family physician with medication, which failed to halt the
progress of her condition. In the week prior to admission there had been increasing pain
and swelling and daily fever.

On examination, she appeared acutely ill with a temperature of 38.9C. There was a
considerable amount of drainage from the ear and granulation tissue was observed on the
floor of the external auditory canal. Gram-negative rods and a few gram-positive cocci in
clumps were noted on Gram stained smear of the drainage fluid.

Her BP was 95/60 mm Hg, pulse 112/min. She had cold extremities, deep sighing
respiration (Kussmauls respiration) and her breath smelt of acetone.

The results of her fasting laboratory investigations were as follows:

Na 130mmol/L ABG
K 5.8mmol/L pH7.05
Bicarbonate 5mmol/L PCO2 2.0kPa
Urea 18mmol/L
Creatinine 140mmol/L
Glucose 32 mmol/L

OBJECTIVES:

1. What are the epidemiological features of diabetes mellitus? List the risk factors.
2. Compare the pathogenesis of NIDDM and IDDM.
3. Explain diabetic keto-acidosis as a complication of diabetes mellitus.
4. Explain the difference between diabetes mellitus and diabetes insipidus.
5. Describe the source(s) of ketone bodies during starvation and in poorly controlled
diabetes.
6. Discuss the biochemical findings shown above.
7. Outline the biochemical complications of longstanding diabetes mellitus.
8. Describe the pathologic changes that can occur in kidney, eyes and cardiovascular
system in longstanding diabetes mellitus.
9. Discuss the monitoring and control of the diabetic patient in a primary care
setting; emphasising preventive strategies e.g. foot care.
10. Discuss the various classes of drugs used in the treatment of Type II diabetes
mellitus.
11. What are the indications for the use of insulin in diabetes mellitus?

Tutors Guidelines:
Diabetes 1 and 2 should be covered in two weeks.

January 7, 2008
GREAT IS 2019. WE PREVAIL.

Note: Diabetic ketosis presents more often in insulin dependants.

January 7, 2008
GREAT IS 2019. WE PREVAIL.

Problem 19. Diabetes (2)

In spite of oral and topical antibiotics the 48-year-old diabetic woman continued to have
high temperatures (38.9C), severe earache and swelling of the pinna and left side of the
face. Cultures reveal Pseudomonas aeruginosa susceptible to piperacillin, gentamicin,
tobramycin and ciprofloxacin and resistant to other antimicrobial agents and
Staphylococcus aureus, resistant to penicillin G but sensitive to penicillinase-resistant
penicillins.

OBJECTIVES:

1. Discuss the primary care approach to infections in diabetes, emphasising the


predisposition to infections in diabetics.
2. List aetiological agents of external otitis.
3. What are the complications of malignant external otitis?
4. Describe the mechanism of action of penicillins, aminoglycosides and
fluoroquinolones.
5. Describe what is meant by penicillinase-resistant penicillins and give examples.

January 7, 2008
GREAT IS 2019. WE PREVAIL.

Problem 20. Goitre

DD, a 49-year old woman presented to clinic with a year long history of feeling
increasingly hot, with diarrhoea, and an increased appetite, but with weight loss of over
25lbs (11.4kg). She also complained of palpitations. As she sat before the consultant,
she continually scratched her skin and shifted from side to side in her chair. On closer
examination, she was noted to have a diffuse goitre with a bruit, lid lag and lid retraction
but no exophthalmos. Her temperature was recorded at 370C, pulse 94/minute, BP140/80
mm.Hg and weight 55kg. Her chest was clear and no other abnormality was found Blood
investigations were requested and confirmed the clinical diagnosis: Free T4: 36
micrograms/dl (normal range 4.5-12.0); TSH: 0.02 mU/L (normal range 0.47 5.01).
She was started on carbimazole and propanolol. However, since these failed to
adequately control her symptoms, she was treated with radioactive iodine (I131).

At the follow-up visit, two months after radioactive iodine treatment, she reported feeling
much better with far less itching, palpitations and sweating. Also, she had started to put
on weight. Repeat thyroid function test results showed that her free T4 concentration had
fallen to 8.6 micrograms/dl.

However, four months later, she complained to the clinic doctor that she was
experiencing occasional constipation, cramps and numbness in her hands and feet. She
was still putting on weight and found that her face was looking somewhat puffy. Her
pulse was recorded as 60/minute, BP131/87 mm.Hg and weight 64.2kg. Repeated
thyroid function tests showed evidence of hypothyroidism secondary to radioactive
iodine ablation.

T4: 0.5 micrograms/dl (NR 4.5 12.0):


T3: 70.0 nanograms/dl (NR 86.0 187.0);
TSH: 65.69 mU/L (NR 0.47 5.01)

She was started on L-thyroxine, 0.1mg daily.

At revisit three months later, she reported feeling very well. The facial puffiness and
cramps had disappeared. Her pulse was found to be 71/min. Repeated thyroid function
tests confirmed her euthyroid state:

T4: 7.2 micrograms/dl (NR 4.5 12.0)


T3: 131.0 nanograms/dl (NR 86.0 187.0)
TSH: 0.93 mU/L (NR 0.47 1.85)

Since then, she has been maintained on thyroxine 0.1mg daily and remains stable.

OBJECTIVES:

1. Describe the hypothalamus-pituitary-thyroid axis.


2. Discuss the importance of iodine in the pathogenesis of thyroid diseases.
3. Describe the immunopathogenesis of Graves disease.

January 7, 2008
GREAT IS 2019. WE PREVAIL.

4. Compare and contrast the pathogenic mechanisms and clinical features of Graves
disease, Hashimotos thyroiditis, and primary myxodema.
5. Explain the difference between hyperthyoidism and T3-toxicosis.
Discuss thyroid function tests.in Graves disease.
6. Outline the changes present in tissues such as heart, skeletal muscle and skin in
Graves disease.
7. Discuss the synthesis of thyroid hormones and the sites of action of antithyroid
drugs.
8. List the drugs used in the treatment of hyperthyroidism and indicate their sites of
action; list their associated toxicities.
9. Explain the role of beta blockers in Graves Disease.

Tutors guidelines: Emphasize negative feedback mechanisms.

January 7, 2008

S-ar putea să vă placă și